Neurology Exam Questions Flashcards

1
Q

Which of the following anticonvulsants is most likely to decrease the half-life of lamotrigine in a clinically significant manner?

A. Carbamazepine.

B. Clonazepam.

C. Gabapentin.

D. Sodium valproate.

E. Vigabatrin.

A

A. Carbamazepine

Carbamazepine: Lamotrigine may enhance the adverse/toxic effect of Carbamazepine. Carbamazepine may increase the metabolism of Lamotrigine. Carbamazepine administration can decrease serum lamotrigine concentrations by approximately 40%.5 Such is likely due to carbamazepine induction of CYP isoenzymes responsible for lamotrigine metabolism.

Sodium Valproate: May enhance the adverse/toxic effect of Lamotrigine and may increase the serum concentration of Lamotrigine.

How well did you know this?
1
Not at all
2
3
4
5
Perfectly
2
Q

An eight-year-old boy presents to the emergency department with a three day history of double vision.
Examination reveals that the two images are most separated when looking to the right. When looking to the right with the left eye covered, the more medial image disappears.

The nerve involved is the:
A. left abducens.
B. left oculomotor.
C. left trochlear.
D. right abducens.
E. right oculomotor.

A

D. right abducens.

Binocular diplopia is present with both eyes open and absent when either eye is closed. Binocular diplopia reflects conditions in which the visual axes are misaligned. In general, most patients will close the eye with the dysfunctional muscle unless that is the eye with the much better vision.

The orbital muscles are innervated by cranial nerve III except for the superior oblique (cranial nerve IV) and the lateral rectus (cranial nerve VI) muscles. Any condition that results in palsy of the third, fourth, or sixth cranial nerves can cause binocular diplopia.
When the eyes fix on an image, impairment in the movement of one eye results in projection of the image upon the macular area in the normal eye and to one side of the macula in the paretic eye and thus 2 images are perceived. The image seen by the paretic eye (false image) is ALWAYS outermost. For this boy, diplopia is maximal on looking to the right, and the medial (true) image disappears when covering the left eye. This leaves only the false image. Thus, the abnormality is on abduction of the right eye, initiated by the lateral rectus muscle and innervated by the Right abducens (6th) nerve.

A: Left abducens - Lateral rectus. Patients complain of horizontal diplopia. There is esotropia (inward deviation) of the left eye, worse with gaze into the field of the weak lateral rectus muscle (i.e. to the left). Patients assume a compensatory face turn in the direction of the paralysed muscle. Abduction is commonly limited on the side of the lesion. The patient would complain of diplopia on left lateral gaze, and the medial (true) image would disappear on covering the right eye.

B: Left Occulomotor - may cause both horizontal and vertical diplopia. If complete palsy, patients have ptosis and large unreactive pupil, paralysis of adduction, upward and downward gaze. The eye rests in the position of abduction, depression and intorsion (medial rotation of upper pole). The left eye deviates laterally in the resting position due to unopposed action of the intact left lateral rectus muscle.

C: Left Trochlear - Superior Oblique. Common cause of vertical diplopia, most palsies being traumatic or idiopathic. The eyes appear conjugate in the primary position, but testing eye movements reveals defective depression of the left eye when adducted (i.e. patient looks to right and down, left eye is adducted but does not look down. More often patients complain of diplopia on looking downwards (when descending stairs or reading) and the head may tilt to the side opposite the weak superior oblique (i.e. to the right) to minimise the diplopia.

D: Right abducens - controls lateral rectus muscle of right eye, thus preventing abduction of this eye. When looking toward the field of the weak muscle (i.e. the right), diplopia is greatest. The eyes appear conjugate in the primary position. Diplopia is horizontal (true and fake image side by side) and is present only when looking to the paralysed side and maximal at the extreme of binocular lateral vision.

E: Right Occulomotor - As for answer B, but involves the right eye

How well did you know this?
1
Not at all
2
3
4
5
Perfectly
2
Q

A six-year-old boy presents with a six-month history of swallowing problems and speech change. His mother says that his speech is less clear and that fluids occasionally come out of his nose when he is drinking.
A photograph of his mouth is shown below.

What is the most likely diagnosis?
A. Anterior horn cell disorder.
B. Arnold-Chiari malformation.
C. Cervical cord tumour.
D. Myasthenia gravis.
E. Myotonic dystrophy.

A

A. Anterior Horn Cell Disorder

Anterior Horn Cell Disorders
Include Charcot-Marie-Tooth, SMA, poliomyelitis, amyotrophic lateral sclerosis (most common). Anterior Horn is the ventral grey matter of the spinal cord that contains motor neurons that affect axial muscles.

ALS
The clinical hallmark of amyotrophic lateral sclerosis (ALS) is the combination of upper motor neuron and lower motor neuron signs and symptoms. Upper motor neuron findings of weakness, hyperreflexia, and spasticity result from degeneration of frontal motor neurons. The lower motor neuron findings of weakness, atrophy or amyotrophy, and fasciculations are a direct consequence of degeneration of lower motor neurons in the brainstem and spinal cord.

The initial clinical manifestation of ALS may occur in any body segment (bulbar, cervical, thoracic or lumbosacral) and may manifest as upper motor neuron or lower motor neuron symptoms or signs. Twenty percent of patients will have onset in the bulbar segment, which most often presents with either dysarthria or dysphagia.

Loss of UMNs results in slowness of movement, incoordination and stiffness with relatively little overt weakness. Arm or hand UMN symptoms include poor dexterity with resulting difficulty performing activities of daily living. Leg UMN symptoms manifest as a spastic gait with poor balance and may include spontaneous leg flexor spasms and ankle clonus.

Dysarthria and dysphagia are the most common bulbar UMN symptoms. UMN or spastic dysarthria produces a characteristically strained vocal quality with slow speech. UMN dysphagia results from slow and discoordinated contraction of the swallowing muscles, which may lead to coughing and choking. Another frequent bulbar UMN symptom is the syndrome of the pseudobulbar affect. This is manifested as inappropriate laughing, crying, or yawning.

Loss of LMNs results in weakness, usually accompanied by atrophy and fasciculations. Cramps are also common.
Hand weakness causes difficulty manipulating small objects (buttons, zippers, coins) and using writing instruments. Proximal arm weakness results in difficulty elevating the arm to the level of the mouth or above the head. This can produce difficulty with bathing, dressing, grooming and eating. Foot and ankle weakness results in tripping, a slapping gait, and falling. Proximal leg weakness results in difficulty arising from chairs, climbing stairs and getting off of the floor. Balance may also be adversely affected.

Dysarthria and dysphagia can also result from LMN damage. Dysarthria may result from weakness of the tongue, lips or palate. The speech is usually slurred and may have a nasal quality. Hoarseness may be caused by associated vocal cord weakness. Dysphagia results from tongue weakness with disruption of the oral phase of swallowing or from pharyngeal constrictor weakness with disruption of the pharyngeal phase of swallowing or both. Tongue weakness may lead to pocketing of food between the cheeks and gums. Pharyngeal weakness often manifests as coughing and choking on food, liquids or secretions such as saliva or mucus. Aspiration may result.

SMAs are degenerative diseases of motor neurons that begin in fetal life and continue to be progressive in infancy and childhood. The progressive denervation of muscle is compensated in part by reinnervation from an adjacent motor unit, but giant motor units are thus created with subsequent atrophy of muscle fibers when the reinnervating motor neuron eventually becomes involved. Upper motor neurons remain normal.

Type 0 - severe fetal, usually fatal in early neonatal period.

Type1 - severe infantile. The cardinal features are severe hypotonia; generalized weakness; thin muscle mass; absent tendon stretch reflexes; involvement of the tongue, face, and jaw muscles; and sparing of extraocular muscles and sphincters.

Type 2 - late infantile. Affected infants are usually able to suck and swallow, and respiration is adequate in early infancy. These children show progressive weakness, but many survive into the school years or beyond, although confined to an electric wheelchair and severely handicapped. Nasal speech and problems with deglutition develop later. Scoliosis becomes a major complication in many patients with long survival.

Type 3 - juvenile. Can appear normal in infancy. The progressive weakness is proximal in distribution, particularly involving shoulder girdle muscles. Patients are ambulatory. Symptoms of bulbar muscle weakness are rare. About 25% of patients with this form of SMA have muscular hypertrophy rather than atrophy, and it may easily be confused with a muscular dystrophy. Longevity can extend well into middle adult life. Fasciculations are a specific clinical sign of denervation of muscle. In thin children, they may be seen in the deltoid, biceps brachii, and occasionally the quadriceps femoris muscles, but the continuous, involuntary, wormlike movements may be masked by a thick pad of subcutaneous fat. Fasciculations are best observed in the tongue, where almost no subcutaneous connective tissue separates the muscular layer from the epithelium.

Arnold Chiari Malformation

Chiari malformations are a heterogeneous group of disorders that are defined by anatomic anomalies of the cerebellum, brainstem, and craniocervical junction, with downward displacement of the cerebellum, either alone or together with the lower medulla, into the spinal canal.

Chiari I malformation (CM-I) is characterized by abnormally shaped cerebellar tonsils that are displaced below the level of the foramen magnum

Chiari II malformation (CM-II), also known as Arnold-Chiari malformation, is characterized by downward displacement of the cerebellar vermis and tonsils, a brainstem malformation with beaked midbrain on neuroimaging, and a spinal myelomeningocele

Chiari III malformation (CM-III) is rare and combines a small posterior fossa with a high cervical or occipital encephalocele, usually with displacement of cerebellar structures into the encephalocele, and often with inferior displacement of the brainstem into the spinal canal

Chiari IV malformation (CM-IV) is now considered to be an obsolete term that describes cerebellar hypoplasia unrelated to the other Chiari malformations

Because it is nearly always associated with a lumbosacral or thoracic myelomeningocele, CM-II is usually detected prenatally or at birth . Manifestations in infancy may include dysphagia, arm weakness, stridor, apneic spells, and aspiration. In late infancy and childhood, progressive hydrocephalus is a common problem in CM-II. In addition, CM-II may be associated with one or more of the syndromes associated with CM-I, such as syringomyelia and scoliosis.

Cervical Cord Tumour
Both benign and malignant tumors can produce a myelopathy as a result of external compression or intramedullary growth.

The most common syndrome is that of extradural spinal cord compression, as produced by metastases to the extradural space. Patients present with a progressive weakness below the level of the lesion with accompanying sensory loss and bladder dysfunction. Pain at the site of involvement is typical. Progression to paraplegia can occur abruptly, as a result of vascular compression.

Myasthenia Gravis
Two forms, occular and generalised.
In ocular myasthenia, the weakness is limited to the eyelids and extraocular muscles.
In generalized disease, the weakness commonly affects ocular muscles, but it also involves a variable combination of bulbar, limb, and respiratory muscles.

More than 50 percent of patients present with ocular symptoms of ptosis and/or diplopia. Of those who present with ocular manifestations, about half will develop generalized disease within two years. Many of the patients who present without ocular manifestations develop ptosis or diplopia at some point in the course.

About 15 percent of patients present with bulbar symptoms. These include dysarthria, dysphagia, and fatigable chewing.

Less than 5 percent present with proximal limb weakness alone.

Bulbar Muscles - Muscles of jaw closure are often involved and produce weakness with prolonged chewing (fatigable chewing). The patient frequently notes that this occurs half-way through a meal, especially when chewing something difficult like steak. When jaw weakness is present at rest, the patients often use their fingers under the jaw to keep the mouth shut.

Oropharyngeal muscle weakness produces dysarthria and dysphagia. The quality of speech sounds nasal when there is weakness of the palatal muscles, or it may be of low intensity (hypophonic). These symptoms often worsen with prolonged speech. Dysphagia may be prominent; the patient may be unable to swallow medications or consume adequate food or liquids. Imminent risk of aspiration may produce a “myasthenic crisis”. Nasal regurgitation, particularly of liquids, may occur due to palatal weakness.

Myotonic Dystrophy
Myotonic dystrophy type 1 (DM1) and myotonic dystrophy type 2 (DM2) are multisystem disorders characterized by skeletal muscle weakness and myotonia, cardiac conduction abnormalities, cataracts, testicular failure, hypogammaglobulinemia, impaired sleep and excessive daytime sleepiness, and insulin resistance.
The symptoms of this childhood form of DM1, with onset of symptoms before the age of 12, typically reflect the involvement of systems and organs other than skeletal muscle; these include cognitive deficiency, difficulty with speech (dysarthria) and hearing, poor coordination, and rarely, postoperative apnea.
Serious cardiac rhythm disturbances may occur in asymptomatic adolescents with no or only subtle signs of myotonic dystrophy. Sports and physical exercise precipitate arrhythmias in over one-half of these patients. Fewer than 10 percent of patients have clinical evidence of a cardiomyopathy and heart failure.

Up to Date and Nelsons

How well did you know this?
1
Not at all
2
3
4
5
Perfectly
3
Q

An otherwise normal five-year-old child is brought to see you because of recurrent generalised convulsions. The child had his first febrile convulsion at 12 months of age, and subsequently has had six afebrile seizures. He is on anticonvulsant therapy. There is no family history of febrile convulsions or epilepsy.
His mother asks about risk for sudden, unexplained [unexpected] death due to epilepsy (SUDEP).
Which of the following is the greatest risk factor for SUDEP?
A. Idiopathic epilepsy.
B. Male sex.
C. Neurological deficit.
D. Symptomatic epilepsy.
E. Young age.

A

C. Neurological deficit.
Risk factors for SUDEP include early age of epilepsy onset, frequent generalized tonic-clonic seizures, and intractable epilepsy. Case-control and cohort studies of SUDEP have identified certain clinical and demographic features as potential risk factors, although these are not all consistently found in all studies:

  • Seizure frequency (>1/month)
  • Medication noncompliance, subtherapeutic AED level
  • Age 20 to 45 years
  • Generalized tonic-clonic seizures
  • Polytherapy
  • Duration of epilepsy (>10 years)
  • Alcoholism
  • Male gender
  • Nocturnal seizures
How well did you know this?
1
Not at all
2
3
4
5
Perfectly
4
Q

The diagnosis of neurofibromatosis type 1 (NF-1) is made in a seven-year-old boy. He is developmentally normal. Of the following, which complication of NF-1 is he most likely to develop during childhood or adolescence?
A. Hypertension.
B. Malignant transformation of a neurofibroma.
C. Scoliosis.
D. Seizures.
E. Sensorineural deafness.

A

C. Scoliosis

The typical order of appearance of clinical manifestations is café-au-lait macules, axillary and/or inguinal freckling, Lisch nodules (iris hamartomas), and neurofibromas. Osseous lesions, if present, usually appear during the patient’s first year after birth, and symptomatic optic pathway glioma (OPG) usually occurs by the time the patient is three years of age . Other tumors and neurologic complications typically begin to appear after the first year of life, and hypertension and malignant transformation of tumors may occur in adolescence and adulthood. Scoliosis is a common complication found in about 10% of the patients.

How well did you know this?
1
Not at all
2
3
4
5
Perfectly
5
Q

Neuroadaptation (tolerance) syndrome is the phenomenon of loss of treatment response generally after months of a clear and definite response to medication. With which of the following medications is this most likely to occur in children?
A. Carbamazepine.
B. Clonazepam.
C. Fluoxetine.
D. Methylphenidate.
E. Sodium valproate.

A

B. Clonazepam
Tolerance to the depressant effects of benzodiazepines is rapid, but tolerance to the anxiolytic effects develops slowly and to a limited extent.

How well did you know this?
1
Not at all
2
3
4
5
Perfectly
5
Q

An otherwise normal six-year-old boy is diagnosed with moderate sensorineural hearing loss. There is
no significant family history.
The most likely cause of his hearing loss is:
A. congenital cytomegalovirus.
B. congenital rubella.
C. Connexin 26 mutation.
D. Pendred syndrome.
E. Usher syndrome.

A

C. Connexin 26 Mutation

Connexin 26 mutation
Mutations of the connexin-26 and -30 genes have been identified in autosomal recessive (DNFB 1) and autosomal dominant (DNFA 3) SNHL and in sporadic patients with nonsyndromic SNHL; up to 50% of nonsyndromic SNHL may be related to a mutation of connexin-26.

Congenital Cytomegalovirus
The most common infectious cause of congenital SNHL is cytomegalovirus (CMV), which infects 1/100 newborns in the USA. Of these, 6,000-8,000 infants each year have clinical manifestations, including approximately 75% with SNHL. Congenital CMV warrants special attention because it is associated with hearing loss in its symptomatic and asymptomatic forms, and the hearing loss may be progressive. Some children with congenital CMV have suddenly lost residual hearing at 4-5 yr of age.

Congenital Rubella
Rubella, once the most common viral cause of congenital SNHL, is very uncommon because of effective vaccination programs.

Pendred and Usher Syndromes
Autosomal recessive genetic SNHL, both syndromic and nonsyndromic, accounts for about 80% of all childhood cases of SNHL. Usher syndrome (types 1, 2, and 3), Pendred syndrome, and the Jervell and Lange-Nielsen syndrome (one form of the long Q-T syndrome) are 3 of the most common syndromic recessive types of SNHL. Usher Syndrome also involves retinitis pigmentosa and impaired vestibular function. Pendred Syndrome involves SNHL and goitre.

How well did you know this?
1
Not at all
2
3
4
5
Perfectly
5
Q

A ten-year-old boy is investigated for excessive day time sleepiness. He sleeps 11 hours per night and
snores most nights. He wakes frequently at night and calls out to his mother. He describes periods of being not being able to move in bed as though he is paralysed. He
occasionally falls to the ground when laughing.

The most likely diagnosis is:
A. atonic epilepsy.
B. narcolepsy.
C. nocturnal epilepsy.
D. obstructive sleep apnoea.
E. parasomnia.

A

B. Narcolepsy

Narcolepsy
Narcolepsy is characterized by excessive daytime sleepiness, cataplexy (sudden loss of tone), sleep paralysis, hypnogogic hallucinations, and disturbed nighttime sleep. Loss of tone occurs in response to strong emotions, and spreads from the face downwards. Consciousness is maintained in cataplexy. A selective loss of hypocretin-secreting neurons in the hypothalamus is at the origin of this disorder. The fact that DQB1*0602 is a predisposing HLA allele identified in 85-95% of patients with narcolepsy-cataplexy suggests an autoimmune mediated neuronal loss. Diagnosis is based on the multiple sleep latency test, and therapy relies on scheduled naps, amphetamines, methylphenidate, tricyclic antidepressants, and counseling about precautions in work and driving.

Atonic Epilepsy
Atonic seizures are characterized by flaccidity or lack of movement during a convulsion. Atonic seizures, are usually longer and the loss of tone often develops more slowly. Sometimes it is difficult to distinguish among tonic, myoclonic, atonic, or astatic seizures based on the history alone when the family reports only that the patient “falls”; in such cases, the seizure may be described as a drop attack. A mechanistically similar seizure can involve the tone of only the head and neck; this seizure morphology is referred to as a head drop. Tonic, clonic, myoclonic, and atonic seizures can be focal (including one limb or one side only), focal with secondary generalization, or primary generalized.

Nocturnal Epilepsy
Benign childhood epilepsy with centrotemporal spikes (BECTS) which typically starts during childhood and is outgrown in adolescence. The child typically wakes up at night owing to a simple partial seizure causing buccal and throat tingling and tonic or clonic contractions of one side of the face, with drooling and inability to speak but with preserved consciousness and comprehension. Complex partial and secondary generalized seizures can also occur. EEG shows typical broad-based centrotemporal spikes that are markedly increased in frequency during drowsiness and sleep. MRI is normal. Patients respond very well to AEDs such as carbamazepine. In some patients who only have rare and mild seizures treatment might not be needed.

Obstructive Sleep Apnoea
A respiratory disorder that is characterized by repeated episodes of prolonged upper airway obstruction during sleep despite continued or increased respiratory effort, resulting in complete (apnea) or partial (hypopnea; ≥50% reduction in airflow) cessation of airflow at the nose and/or mouth, as well as in disrupted sleep. Both intermittent hypoxia and the multiple arousals resulting from these obstructive events likely contribute to significant metabolic, cardiovascular, and neurocognitive/neurobehavioral morbidity.

Parasomnia
Parasomnias are defined as episodic nocturnal behaviors that often involve cognitive disorientation and autonomic and skeletal muscle disturbance. Parasomnias may be further characterized as occurring primarily during NREM sleep (partial arousal parasomnias) or in association with REM sleep, including nightmares, hypnogogic hallucinations, and sleep paralysis; other common parasomnias include sleep-talking. Partial arousal parasomnias, which include sleepwalking, sleep terrors, and confusional arousals are more common in preschool and school-aged children because of the relatively higher percentage of SWS in younger children. Any factor that is associated with an increase in the relative percentage of SWS (certain medications, previous sleep deprivation) may increase the frequency of events in a predisposed child. There appears to be a genetic predisposition for both sleepwalking and night terrors. In contrast, nightmares, which are much more common than the partial arousal parasomnias but are often confused with them, are concentrated in the last third of the night, when REM sleep is most prominent. Partial arousal parasomnias may also be difficult to distinguish from nocturnal seizures.

The partial arousal parasomnias have several features in common. Because they typically occur at the transition out of “deep” or SWS, partial arousal parasomnias have clinical features of both the awake (ambulation, vocalizations) and the sleeping (high arousal threshold, unresponsiveness to the environment) states; there is usually amnesia for the events. The typical timing of partial arousal parasomnias during the first few hours of sleep is related to the predominance of SWS in the first third of the night; the duration is typically a few minutes (sleep terrors) to an hour (confusional arousals). Sleep terrors are sudden in onset and characteristically involve a high degree of autonomic arousal (i.e., tachycardia, dilated pupils), while confusional arousals typically arise more gradually from sleep, may involve thrashing around but usually not displacement from bed, and are often accompanied by slow mentation on arousal from sleep (“sleep inertia”). Sleepwalking may be associated with safety concerns (e.g., falling out of windows, wandering outside). Avoidance of, or increased agitation with, comforting by parents or attempts at awakening are also common features of all partial arousal parasomnias.

Nelsons

How well did you know this?
1
Not at all
2
3
4
5
Perfectly
6
Q

A nine-month-old child is reviewed because of concerns about the appearance of his face. He
suffered birth trauma related to shoulder dystocia. A brachial plexus injury was identified at birth.
What level of the brachial plexus has been injured to explain the facial findings?
A. C5.
B. C6.
C. C7.
D. C8.
E. T1.

A

E. T1

Klumpke paralysis is a rare form of brachial palsy, in which injury to the 7th and 8th cervical nerves and the 1st thoracic nerve produces a paralyzed hand and ipsilateral ptosis and miosis (Horner syndrome) if the sympathetic fibers of the 1st thoracic root are also injured. Mild cases may not be detected immediately after birth. Differentiation must be made from cerebral injury; from fracture, dislocation, or epiphyseal separation of the humerus; and from fracture of the clavicle. MRI demonstrates nerve root rupture or avulsion.

How well did you know this?
1
Not at all
2
3
4
5
Perfectly
6
Q

A nine-month-old girl is referred by her general practitioner because of recurrent events occurring on a daily basis over the last two weeks. The episodes are stereotyped and consist of her stopping what she is doing, flexing at her trunk, and pressing her hands above her inguinal region. There is associated tremulousness and jaw rigidity. The events last one to two minutes with her becoming red in the face and grunting. She seems to be preoccupied and gets distressed if touched or moved. After the event, the child goes to sleep. The events never occur in sleep.
The most likely diagnosis is:
A. dystonia.
B. frontal lobe seizures.
C. gastroesophageal reflux.
D. infantilemasturbation.
E. urinary infection.

A

D. Infantile masturbation

Infantile Masturbation
Very common in childhood. Peak at approx 4 years. In infancy and early childhood, usually practiced by rubbing thighs together, rhythmic body movement and pelvic thrusts. Child may appear to be in a trance with a flushed face, glassy stare and audible breathing. If orgasm occurs, may be followed by general relaxation, pallor and sweating. Child may appear exhausted and fall asleep. (Childhood Masturbation. Clinical Paediatrics, 1993).

Dystonia
Dystonia is a movement disorder characterized by involuntary, sustained muscle contractions that result in twisting and repetitive movements or abnormal postures. The clinical features of dystonia vary according to anatomic location, age of onset, and etiology. Early onset dystonia usually begins in a leg, often as intorsion of the foot. Spread from one leg to other body areas occurs in approximately 50 to 90 percent of children, usually within five years of onset. Late onset primary focal dystonia may involve different body areas.

Frontal Lobe Seizures
Frontal lobe seizures often occur at night and can be very numerous and brief (

Gastroesophageal Reflux
GER is common in infants and usually is not pathological. Regurgitation is present in 50 to 70 percent of all infants, peaks at age four months, and typically resolves by one year. A small minority of infants with GER develop other symptoms suggestive of GERD, including feeding refusal, irritability, hematemesis, anemia, respiratory symptoms, and failure to thrive.

Urinary Infection
In a meta-analysis of the diagnostic accuracy of the symptoms and signs of UTI in children younger than two years, the following symptoms and signs were the most helpful in identifying children with UTI:

  • History of UTI (likelihood ratio [LR] 2.3)
  • Temperature >40ºC (LR 3.2)
  • Suprapubic tenderness (summary LR 4.4)
  • Lack of circumcision (summary LR 2.8)

Up to Date, Nelsons

How well did you know this?
1
Not at all
2
3
4
5
Perfectly
7
Q

A 21-month-old girl presents, with her siblings, with a viral upper respiratory tract infection. Her facial features are noted to be different from her siblings and this has been present since birth. A photograph is shown (the patient is seated in the middle). She has no history of feeding or breathing difficulties as an infant. On examination her eye movements are normal with normal pupillary responses. Her smile is equal and symmetrical. What is the most likely diagnosis?

A. Bilateral facial nerve (VII) palsy.
B. Bilateral oculomotor (III) nerve palsy.
C. Congenital myasthenia gravis.
D. Congenital ptosis.
E. Möbius syndrome.

A

D. Congenital ptosis
Congenital Ptosis
Congenital ptosis is most often associated with absence or reduction of striated levator palpebrae superioris muscle. Müller’s muscle remains relatively intact.

Congenital ptosis is usually unilateral (in 69 to 75 percent), neurologically isolated, and nonprogressive. There can be a familial association, but it may be unrecognized if family members are only mildly affected. Levator function (LF) varies in proportion to the degree of ptosis. Lid creases are often absent or lower than normal. Other ophthalmologic findings may include amblyopia and strabismus in 20 to 30 percent.

Bilateral Facial Nerve Palsy
Extremely rare. Congenital cause is usually Mobius syndrome. Unilateral due to birth injury more common.

Bilateral Oculomotor nerve palsy
The third cranial nerve supplies the levator muscle of the eyelid and four extraocular muscles: the medial rectus, superior rectus, inferior rectus, and inferior oblique. The superior oblique muscle is innervated by cranial nerve IV and the lateral rectus muscle by cranial nerve VI. In addition, the third cranial nerve constricts the pupil through its parasympathetic fibers that supply the smooth muscle of the ciliary body and the sphincter of the iris. Children with congenital third nerve palsies may not complain of diplopia because they ignore or suppress the second image or because they have superimposed amblyopia; often they are brought to medical attention by their parents, who have noticed ptosis or strabismus.

Congenital Myasthenia Gravis
Newborns with congenital myasthenia frequently have ptosis, in contrast to patients with the transient disorder. In addition, they typically demonstrate ophthalmoplegia and bulbar and respiratory muscle weakness. Affected infants may have fluctuating generalized hypotonia and weakness and life-threatening episodes of apnea. Arthrogryposis may be present at birth

Mobius syndrome

The distinctive features of Möbius syndrome are congenital facial paresis and abduction weakness. The facial palsy is commonly bilateral, often asymmetric, and often incomplete, tending to spare the lower face and platysma. Ectropion, epiphora, and exposure keratopathy can develop. The abduction defect may be unilateral or bilateral. Esotropia is common.

How well did you know this?
1
Not at all
2
3
4
5
Perfectly
8
Q

A 12-year-old boy presents with a two year history of tremor predominantly affecting his hands. The tremor is worse when he is anxious, upset or active. There is a family history of tremor in a paternal grandparent.
On examination, the tremor is present when his hands are outstretched in front of him. The tremor worsens when asked to perform finger-to-nose movements. Examination is otherwise normal, including gait.

The most likely diagnosis is:
A. anxiety.
B. cerebellar tremor.
C. dystonia.
D. essential tremor.
E. Wilson disease.

A

D. essential tremor.

Tremor is a rhythmic, oscillatory movement around a central point or plane, which results from the action of antagonist muscles. Tremor can affect the extremities, head, trunk, or voice and can be classified by both its frequency (slow [4 Hz], intermediate [4-7 Hz], and fast [>7 Hz]) and by the context in which it is most pronounced. Rest tremor is maximal when the affected body part is inactive and supported against gravity, whereas postural tremor is most notable when the patient sustains a position against gravity. Action tremor occurs with performance of a voluntary activity and can be subclassified into simple kinetic tremor, which occurs with limb movement, and intention tremor, which occurs as the patient’s limb approaches a target and is a feature of cerebellar disease.

Essential tremor
Autosomal dominant condition.
Essential tremor is characterized by a slowly progressive, bilateral, 4-9 Hz postural tremor that involves the upper extremities and occurs in the absence of other known causes of tremor. Mild asymmetry is common, but ET is rarely unilateral. ET may be worsened by actions, such as trying to pour water from cup to cup, and affected adults may report a history of ethanol responsiveness. Most young children present to care because a parent, teacher, or physician has noticed the tremor, rather than because the tremor is causing impairment. That being the case, most children with ET do not require pharmacologic intervention. If they are having difficulty with their handwriting or self-feeding, an occupational therapy evaluation and/or assistive devices, such as wrist weights and weighted silverware, may be helpful. Teenagers tend to report more impairment from ET; however, it is unclear whether this is due to actual or subjective progression of the tremor. Teenagers who do require pharmacotherapy usually respond to the same medications that are used in adults—propranolol and primidone. Propranolol, which is generally considered the first-line treatment.
Some patients with ET develop enhanced physiologic tremor due to anxiety or other adrenergic mechanisms, thereby aggravating the underlying tremor. ET is typically relieved by small amounts of alcohol but, in contrast with physiologic tremor, is not usually aggravated by caffeine.

Anxiety
These are typically complex resting, postural, and action tremors with abrupt onset, a static course, changeable features, functional disability out of proportion to tremor magnitude, and resistance to treatment. Any body part may be involved, but, remarkably, the fingers are often spared with much of the upper limb tremor occurring at the wrist. Other features of psychogenic movement disorders are also often present, such as inconsistent display of symptoms and clinical features that are incongruous with known tremors.
Examination usually shows variable tremor frequency or tremor entrainment (ie, shift of tremor frequency to the speed of contralateral finger tapping), especially with distraction maneuvers such as repetitive tapping tasks with an uninvolved opposite hand or foot.

Cerebellar Tremor
Cerebellar tremors can be postural, action, or intention (kinetic). In severe cases, they can spill over to also occur at rest. Tremor frequency is typically low (3 to 4 Hz) and can be associated with ataxia and dysmetria.
Rubral tremor is caused by disturbances of cerebellothalamic projections; it is usually present at rest and increases during postural fixation and voluntary activity. Titubation of the head and neck (“to and fro” movements) may be associated with cerebellar tremor; it is distinguished from essential head tremor by the presence of other cerebellar findings.

Dystonia
Dystonia is a movement disorder characterized by involuntary, sustained muscle contractions that result in twisting and repetitive movements or abnormal postures. The clinical features of dystonia vary according to anatomic location, age of onset, and etiology. Early onset dystonia usually begins in a leg, often as intorsion of the foot. Spread from one leg to other body areas occurs in approximately 50 to 90 percent of children, usually within five years of onset. Late onset primary focal dystonia may involve different body areas. Arm dystonia is manifested as a posturing of the hand and/or arm. This problem may be variably present with arms outstretched, but is often not present at rest. Overlying dystonic spasms may occur and resemble essential tremor. However, in contrast to essential tremor, dystonia is often unilateral, and triggered by specific activities, such as writing or typing.

**_Wilson Disease_**
 Wilson disease (hepatolenticular degeneration) is due to a genetic abnormality inherited in an autosomal recessive manner that leads to impairment of cellular copper transport. It is found worldwide, with a prevalence of approximately 1 case in 30,000 live births in most populations. Impaired biliary copper excretion leads to accumulation of copper in several organs, most notably the liver, brain, and cornea. Over time, the liver is progressively damaged and eventually becomes cirrhotic. A small percent of patients develop acute liver failure, most often in the setting of advanced fibrosis of the liver. In addition, patients may develop neurologic complications, which can be severe. The clinical manifestations of Wilson disease are predominantly hepatic, neurologic, and psychiatric, with many patients having a combination of symptoms. Hemolysis is also a common finding in patients with acute liver failure due to Wilson disease.

Tremors in Wilson disease may occur at rest or with action and may have multiple position and task-dependent characteristics. Some of the tremors seen include:

  • A tremor that resembles essential tremor (variable amplitude and frequency, with arm, and sometimes head and leg involvement). Unlike essential tremor, the tremor in patients with Wilson disease often persists in its asymmetry and may not involve the voice.
  • An intention (kinetic) tremor (low amplitude, medium-to-high frequency), seen most often in a distal upper extremity. Intention tremors typically increase in severity as the patient’s hand moves closer to its target.
  • A postural tremor that occurs when the patient assumes a particular position. The classic tremor associated with Wilson disease is a wing-beating tremor, though it is not the most common type of tremor. The tremor is a low frequency, high amplitude tremor that is most prominent when the patient’s arms are held outstretched laterally or with the arms extended in front of the patient with the palms facing downward and the elbows flexed. The tremor increases in amplitude with increased duration of posture holding. The name comes from the fact that the movement of the patient’s arms may be reminiscent of a bird’s flapping wings.
  • A unilateral, rest tremor may be present, but is rarely seen in isolation. When present, it is typically accompanied by postural and intention tremors that often dominate the clinical picture.
How well did you know this?
1
Not at all
2
3
4
5
Perfectly
9
Q

A seven-year-old boy presents with a three month history of staring spells associated with eye
flickering and lip smacking movements. His electroencephalogram (EEG) is shown below.

Which of the following anticonvulsants is most likely to aggravate the underlying seizure disorder in
this patient?
A. Carbamazepine.
B. Clonazepam.
C. Ethosuximide.
D. Sodium valproate.
E. Topiramate.

A

A. Carbamazepine.

History is suggestive of absence seizure in which the typical history is onset 5-7 years of age, up to hundreds of events per day lasting a few seconds and accompanied by flutter or upward rolling of eyes. May have simple automatisms such as lip smacking, clothing picking, minimal falling forward of the head. No postictal period and the patient usually resumes what they were doing immediately prior to seizure.
Carbamazepine reported to worsen absence seizures.

Recommended treatment varies amongst organisations. Ethosuxamide, Valproic Acid and Lamotragine are the three variations.

How well did you know this?
1
Not at all
2
3
4
5
Perfectly
11
Q

A seven-year-old boy with a diagnosis of intractable epilepsy is prescribed an anticonvulsant. Six months later, he presents with a four hour history of colicky abdominal pain and vomiting. Examination reveals him to be distressed and pale. He is not jaundiced. During the examination he suffers a bout of pain. A diagnosis of renal colic is made.

Which of the following anticonvulsants is most likely to be associated with this clinical scenario?
A. Carbamazepine.
B. Lamotrigine.
C. Phenytoin.
D. Sodium valproate.
E. Topiramate.

A

E. Topiramate.

  • Carbamazepine - Anemia, agranulocytosis, CNS depression, TENS, Stevens Johnson syndrome, hypersensitivity, hyponatremia, suicidal ideation, activate latent psychosis, confusion/agitation.
  • Lamotrigine - Aseptic meningitis, blood dyscrasias, CNS depression, multiorgan hypersensitivity, severe and potentially life threatening skin rashes (Stevens Johnson, TENS, angiodema).
  • Phenytoin- Blood dyscrasias, decreased bone mineral density, hypotension and severe cardiac arrhythmias with IV infusion (must be done slowly), TENS, Steven-Johnson Syndrome, hypersensitivity, suicidal ideation, gingival hyperplasia, megaloblastic anemia, hyperglycemia.
  • Sodium Valproate - CNS depression, hepatic failure, fulminant hepatitis, hyperammonemia/encephalopathy, hypothermia, pancreatitis, multi organ hypersensitivity, suicidal ideation, thrombocytopenia, spina bifida in pregnancy.
  • Topiramate - CNS effects, Glaucoma, hyperammonemia/encephalopathy, hyperthermia, metabolic acidosis, renal calculus, suicidal ideation.

Renal calculus: Topiramate exhibits weak carbonic anhydrase inhibitory properties and may increase the risk of kidney stones about 2-4 times that of the untreated population. Kidney stones have been reported in children and adults (incidence higher in males). The risk of stones may be reduced by increasing fluid intake.

Up to Date

How well did you know this?
1
Not at all
2
3
4
5
Perfectly
12
Q

A 15-year-old with a history of complex partial seizures is currently on carbamazepine. He develops acute sinusitis and is prescribed an antibiotic by his general practitioner. He presents to the emergency room with depressed conscious state and status epilepticus. He is found to have markedly elevated carbamazepine levels.
Which of the following antibiotics is he likely to have been prescribed?

A. Amoxicillin-clavulinic acid.
B. Cefaclor.
C. Ciprofloxacin.
D. Doxycycline.
E. Erythromycin.

A

E. Erythromycin.

Carbamazepine is a CYP 3A4 substrate as well as an inducer of multiple cytochrome P450 isoenzymes and may be subject to a number of drug-drug interactions. Toxic levels of carbamazepine may result from CYP 3A4 inhibition due to erythromycin, fluoxetine, and cimetidine, among other drugs.

CYP 3A4 inducers, such as phenytoin and phenobarbital, may decrease carbamazepine levels. Plasma levels of many medications, including haloperidol and clozapine, are reduced in the setting of carbamazepine use.

Up to Date

How well did you know this?
1
Not at all
2
3
4
5
Perfectly
12
Q

Which subgroup of children with cerebral palsy is at most risk of hip displacement?

A. Ataxic.
B. Hypotonic.
C. Spastic diplegia.
D. Spastic hemiplegia.
E. Spastic quadriplegia.

A

E. Spastic quadriplegia.

Children who suffer from cerebral palsy and do not walk before the age of 5 have a 58% incidence of hip dislocation (44% bilateral, 14% unilateral). Other factors involved in the causation of hip dislocation include four limb cerebral palsy and tightness of the adductor and iliopsoas muscles with concomitant weakness in the abductor muscles at the hip. (BMJ. 1999 April 17; 318(7190): 1021–1022)

Spastic diplegia is bilateral spasticity of the legs that is greater than in the arms.
Infants with spastic hemiplegia have decreased spontaneous movements on the affected side and show hand preference at a very early age. The arm is often more involved than the leg and difficulty in hand manipulation is obvious by 1 yr of age.
Spastic quadriplegia is the most severe form of CP because of marked motor impairment of all extremities and the high association with mental retardation and seizures.
(Nelsons)

How well did you know this?
1
Not at all
2
3
4
5
Perfectly
14
Q

A six-year-old boy presents to the Emergency Department with acute urinary retention requiring urgent catheterisation. He has been previously well, but his parents state that he has complained of leg and lower abdominal pain for the last three days and has been walking ‘a bit funny’. On examination he is alert and cooperative with normal upper limb power, reflexes and light touch sensation. He has a broad based gait and is unable to arise from a squatting position without assistance. His lower limb reflexes are difficult to elicit and sensation is reduced in a patchy fashion in both lower limbs.

Which of the following is the most likely diagnosis?

A. Conversion disorder.
B. Guillain-Barré syndrome.
C. Multiple sclerosis.
D. Poliomyelitis.
E. Transverse myelitis.

A

Transverse myelitis.

Transverse Myelitis - Rapid development of both motor and sensory deficits. It has multiple causes and tends to occur in 2 distinct contexts. Small children, 3 yr of age and younger, develop spinal cord dysfunction over hours to a few days. They have a history of an infectious disease, usually of viral origin, or of an immunization within the few weeks preceding the 1st development of their neurologic difficulties. The clinical loss of function is often severe and may seem complete. Although a slow recovery is common in these cases, it is likely to be incomplete. The likelihood of independent ambulation in these small children is about 40%. The pathologic findings of perivascular infiltration with mononuclear cells imply an infectious or inflammatory basis. Overt necrosis of spinal cord can be seen.
In older children, the syndrome is somewhat different. Although the onset is also rapid with a nadir in neurologic function occurring between 2 days and 2 wk, recovery is more rapid and more likely to be complete. Pathologic or imaging examination shows acute demyelination.

Guillain-Barre - The paralysis usually follows a nonspecific viral infection by about 10 days. Weakness usually begins in the lower extremities and progressively involves the trunk, the upper limbs, and finally the bulbar muscles, a pattern known as Landry ascending paralysis. Proximal and distal muscles are involved relatively symmetrically, but asymmetry is found in 9% of patients. The onset is gradual and progresses over days or weeks. Particularly in cases with an abrupt onset, tenderness on palpation and pain in muscles is common in the initial stages. Affected children are irritable. Weakness can progress to inability or refusal to walk and later to flaccid tetraplegia. Paresthesias occur in some cases.

Multiple Sclerosis - Multiple sclerosis (MS) is a chronic demyelinating disorder of the brain, spinal cord and optic nerves characterized by a relapsing-remitting course of neurologic episodes separated in time and space. Presenting symptoms in pediatric MS include hemiparesis or paraparesis, unilateral or bilateral optic neuritis, focal sensory loss, ataxia, diplopia, dysarthria, or bowel/bladder dysfunction.

Poliomyelitis - Weakness may vary from one muscle or group of muscles, to quadriplegia, and respiratory failure. Tone is reduced, nearly always in an asymmetric manner. Proximal muscles usually are affected more than distal muscles, and legs more commonly than arms. Reflexes are decreased or absent. The sensory examination is almost always normal.

Weakness typically worsens over two to three days, although sometimes worsening can progress for up to a week. Bulbar involvement occurs in 5 to 35 percent of patients, producing dysphagia, dysarthria, and difficulty handling secretions. There may be encephalitis, usually in infancy. Respiratory insufficiency may occur.

How well did you know this?
1
Not at all
2
3
4
5
Perfectly
15
Q

A six-year-old girl presents to her paediatrician with irritability and unsteadiness. On questioning there is a seven week history of double vision and ataxia. Cranial nerve exam shows a left abducens nerve palsy with mild right facial weakness. She has very brisk reflexes with bilaterally upgoing plantar responses. MRI scan of her brain is performed and shown below.

The MRI is most consistent with:

A. brain abscess.
B. cerebellitis.
C. craniopharyngioma
D. disseminated encephalomyelitis.
glioma.

A

CLINICAL PRESENTATION — Diffuse pontine gliomas can present with varied symptoms depending on the location of the lesion. These include:

Cranial nerve palsies, long tract signs (eg, hemiparesis) and ataxia in over 50 percent of patients. Cranial nerves VI and VII are most commonly affected but III, IV, IX and X may also be involved.
Hydrocephalus with elevated intracranial pressure (ICP) is observed in fewer than 10 percent of patients at presentation.
Intratumoral hemorrhage can be present in about 6 percent of patients at the time of diagnosis [23]. Symptomatic hemorrhage may eventually occur in up to 20 percent of children, usually in necrotic areas of tumor.
None of these symptoms is pathognomonic of diffuse pontine gliomas. Other brainstem tumors can present with similar symptoms.
Cervicomedullary tumors typically present with lower cranial nerve dysfunction and ataxia.
Focal tumors tend to interfere with the cranial nerves adjacent to the site of origin.
Unlike other brainstem tumors, tectal plate lesions present with hydrocephalus in over 90 percent of cases. Although these tumors are rarely biopsied, those that have been studied are almost always low-grade astrocytomas.

How well did you know this?
1
Not at all
2
3
4
5
Perfectly
16
Q

A six-year-old boy is referred for assessment of progressive unsteadiness of gait over the preceding year. Prior to the onset, his mother recalls that he had chicken pox. In other respects he has been well and there is no family history of similar problems. His examination was significant for gait ataxia and dysmetria. A photograph of his eye is shown below.


Which of the following investigations would be expected to be abnormal?

A. Alpha- fetoprotein.
B. Copper and caeruloplasmin.
C. Lysosomal enzymes.
D. Uric acid.
E. Vitamin E.

A

A. Alpha- fetoprotein

The photograph shows oculocutaneous telangiectasias. This, combined with the history of progressive ataxia makes ataxia-telangiectasia most likely.

The telangiectasias are seen on the bulbal conjunctiva, over the bridge of the nose, on the ears and exposed surfaces of the extremities.

Lab findings include elevated levels of alpha feto-protein, increased incidence of chromosome breaks, and decreased IgA, IgG 2 and 4, and IgE levels (in >50% of patients).

Children have a 50-100 times increased risk of lymphoreticular tumours, and death results from infection or tumour dissemination. Ataxia begins approx age 2 and progresses to loss of ambulation by adolescence. Reference: Nelson’s 19th edition, pages 735, 2055.

How well did you know this?
1
Not at all
2
3
4
5
Perfectly
17
Q

A 15-year-old girl presents with a history of deterioration of night vision and peripheral vision. Her past history includes mild global developmental delay, obesity and surgery in infancy for 4-limb post-axial polydactyly. Eye examination reveals a pigmentary retinopathy.
The most likely diagnosis is:

A. autosomal recessive retinitis pigmentosa.
B. bardet-biedl syndrome.
C. choroideraemia.
D. McKusick-kaufman syndrome.
E. refsum disease.

A

B. bardet-biedl syndrome.

Retinitis pigmentosa -

  • progressive degeneration and dysfunction of the retina, primarily affecting photoreceptor and pigment epithelial function.
  • Night and peripheral vision are lost progressively, leading to a constricted visual field and markedly diminished vision in some patients.

*Bardet-Biedl syndrome *

  • autosomal recessive
  • obesity
  • hypogenitalism in men,
  • mental retardation,
  • retinal dystrophy,
  • polydactyly,
  • renal malformations (particularly calyceal abnormalities),
  • hypertension
  • and, over time, progressive chronic kidney disease.

Choroideremia is an x-linked recessive retinal degenerative disease.

McKusick-Kaufman

  • affects the development of the hands and feet, heart, and reproductive system.
  • characterized by a combination of three features:
    • polydactyly
    • heart defects, and
    • genital abnormalities.

Refsum Disease - Peroxisomal disorder. Classic Refsum disease is characterized by the presence of four clinical abnormalities:

  • Retinitis pigmentosa
  • Peripheral polyneuropathy
  • Cerebellar ataxia
  • Elevated cerebrospinal fluid protein concentration (100 to 600 mg/dL) without an increase in cells
How well did you know this?
1
Not at all
2
3
4
5
Perfectly
18
Q

A six-year-old girl presents to the emergency department with a persistent headache and dizziness following a minor fall one week previously in which there was no loss of consciousness. She has had several vomits today and feels unwell. On examination, she is afebrile with a respiratory rate of 24/minute, a heart rate of 110/minute and a blood pressure of 96/55 mmHg. She is pale and quiet but able to respond appropriately to questions and commands. There are no focal neurological signs. Her non-contrast computed tomography (CT) scan of the head is shown below.
Which of the following is the most likely diagnosis?

A. Extradural haematoma.
B. Intracerebral haemorrhage.
C. Meningioma.
D. Subarachnoid haematoma.
E. Subdural haematoma.

A
How well did you know this?
1
Not at all
2
3
4
5
Perfectly
18
Q

A six-month-old girl is referred to the outpatient clinic for assessment of delayed development. She will not reach, sit unsupported or roll. She weighs 7.6 kg (50th-75thpercentile), and measures 67cm in length (50th%). Her head circumference is 41 cm (25th%). Her head and facial appearance is shown below.

The most likely reason for this appearance is:

A. coronal synostosis.
B. deformational plagiocephaly.
C. lambdoid synostosis.
D. metopic synostosis.
E. sagittal synostosis

A

D. metopic synostosis.

How well did you know this?
1
Not at all
2
3
4
5
Perfectly
19
Q

A 36-week-male infant has a focal seizure at two days of age. Investigation of this seizure included the following diffusion weighted magnetic resonance image of the brain on Day three:

Which of the following neurological conditions is this child most likely to develop?

A. Ataxic cerebral palsy.
B. Choreoathetoid cerebral palsy.
C. Spastic diplegia.
D. Spastic hemiplegia.
E. Spastic quadriplegia.

A

D. Spastic hemiplegia.
MRI shows acutely infarcted area. Unilateral lesion in MCA territory, affecting contralateral arm, hand and face. Causes spastic hemiplegia.

How well did you know this?
1
Not at all
2
3
4
5
Perfectly
19
Q

A 13-year-old girl presents to her paediatrician with a four month history of progressive weakness. The weakness is diffuse and she complains of inability to run, and climb stairs. She frequently falls. Her examination shows diffuse 3/5 weakness in her peripheries, absent reflexes with downgoing toes. She has no cranial nerve abnormalities. Sensory examination reveals a symmetrical glove and stocking distribution of altered touch.
Nerve conduction studies show markedly slowed nerve conduction velocities and conduction block.
The most likely diagnosis is:

A. chronic inflammatory demyelinating polyneuropathy.
B. dermatomyositis.
C. Guillain-Barré syndrome.
D. hereditary motor sensory neuropathy.
E. myasthenia gravis.

A

A. Chronic Inflammatory Demyelinating Polyneuropathy

Chronic Inflammatory Demyelinating Polyneuropathy (CIDP)

  • Symmetric
  • Motor involvement > sensory.
  • Weakness of both proximal and distal muscles,

…..this pattern is a hallmark of acquired demyelinating polyneuropathy.
Also…

  • Cranial nerve and bulbar involvement occur in 10 to 20 percent of patients.
  • Globally diminished or absent reflexes
  • Sensory impairment in CIDP is usually greater for vibration and position sense than for pain and temperature sense, reflecting the involvement of larger myelinated fibers.

Dermatomyositis

  • Present with either rash, insidious onset of weakness or both.
  • Rash - 2x classic rashes.
  • The characteristic heliotrope rash is a blue-violet discoloration of the eyelids that may be associated with periorbital edema. Facial erythema crossing the nasolabial folds is also common, in contrast to the malar rash without nasolabial involvement typical of systemic lupus erythematosus.
  • Classic Gottron papules are bright pink or pale, shiny, thickened or atrophic plaques over the proximal interphalangeal joints and distal interphalangeal joints and occasionally on the knees, elbows, small joints of the toes, and ankle malleoli.
  • Weakness is proximal and often insidious and difficult to differentiate from fatigue at onset.
  • Parents may report difficulty climbing stairs, combing hair, and getting out of bed.
  • Examination reveals inability to perform a sit-up, head lag in a child after infancy, and Gower sign (use of hands on thighs to stand from a sitting position). Patients with JDM may roll to the side rather than sit straight up from lying to compensate for truncal weakness.
  • Approximately half of children exhibit muscle tenderness as a result of muscle inflammation.
  • Esophageal and respiratory muscles are also affected, resulting in aspiration or respiratory failure.
  • Evidence of small vessel inflammation is often visible in the nail folds and gums as individual capillary loops that are thickened, tortuous, or absent.
  • Muslce enzymes raised - CK, LDH, AST, ALT.

Guillain-Barré syndrome

  • Postinfectious (10 days post non specific viral infxn) polyneuropathy involving mainly motor but sometimes also sensory and autonomic nerves.
  • Gradual, progressive weakness over days to weeks.
  • Ascending paralysis
  • Weakness can progress to inability or refusal to walk and later to flaccid tetraplegia.
  • Symmetric weakness with diminished or absent reflexes. Minimal loss of sensation occurs, despite paresthesias.Variable initial physical findings can render making an early diagnosis difficult. Examples of less common signs include predominant proximal weakness, hyperreflexia with extensor plantar response, and sphincter disturbances that raise concerns about a spinal cord lesion.

Hereditary Motor Sensory Neuropathy

  • a group of progressive diseases of peripheral nerves. Motor components generally dominate the clinical picture, but sensory and autonomic involvement is expressed later.
  • Charcot-Marie-Tooth Disease is most common. Autosomal dominant.
  • Most patients are asymptomatic until late childhood or early adolescence, but young children sometimes manifest gait disturbance as early as the 2nd year of life. The peroneal and tibial nerves are the earliest and most severely affected. Children with the disorder are often described as being clumsy, falling easily, or tripping over their own feet.
  • Wasting of muscles of anterior compartment of lower leg lead to characteristic stork-like contour.
  • Pes cavus deformities develop due to denervation of intrinsic foot muscles.

Myasthenia Gravis

  • chronic disease characterized by rapid fatigability of striated muscle.
  • Autoimmune disorder which is generally not hereditary, but some familial forms exist.
  • Three clinical varieties are distinguished in childhood: juvenile myasthenia gravis in late infancy and childhood, congenital myasthenia, and transient neonatal myasthenia.
  • Rapid fatigue of muscles is a characteristic feature of myasthenia gravis that distinguishes it from most other neuromuscular diseases.
  • Left untreated, myasthenia gravis is usually progressive and can become life threatening because of respiratory muscle involvement and the risk of aspiration, particularly at times when the child is otherwise unwell with an upper respiratory tract infection. Familial myasthenia gravis usually is not progressive.
How well did you know this?
1
Not at all
2
3
4
5
Perfectly
20
Q

A seven-year-old boy presents with a six month history of episodes occurring in sleep. The events occur in the early morning and wake him. During an episode, he is able to walk into his parent’s room, but is unable to speak. His parents have noticed that during an episode he has “spasms” of the right side of his face with marked salivation. He is able to describe tingling of his tongue after the event.
An electroencephalogram (EEG) is performed and is shown below.

What is the most likely diagnosis?
A. Absence epilepsy.
B. Benign partial epilepsy of childhood.
C. Frontal lobe epilepsy.
D. Parasomnia.
E. Temporal lobe epilepsy.

A

Benign partial epilepsy of childhood.

Absence Seizures

Typical absence seizures usually start at 5-8 yr of age and are often, owing to their brevity, overlooked by parents for many months even though they can occur up to hundreds of times per day. Unlike complex partial seizures they do not have an aura, usually last for only a few seconds, and are accompanied by flutter or upward rolling of the eyes but typically not by automatisms of the complex partial seizure type (absence seizures can have simple automatisms like lip-smacking or picking at clothing and the head can minimally fall forward). Absence seizures do not have a postictal period and are characterized by immediate resumption of what the patient was doing before the seizure. Hyperventilation for 3-5 min can precipitate the seizures and the accompanying 3 Hz spike–and–slow wave discharges. The presence of periorbital, lid, perioral or limb myoclonic jerks with the typical absence usually predicts difficulty in controlling the seizures with medication.
Atypical absence seizures have associated myoclonic components and tone changes of the head and body and are also usually more difficult to treat. They are precipitated by drowsiness and are usually accompanied by 1-2 Hz spike–and–slow wave discharges.
Juvenile absence seizures are similar to typical absences but occur at a later age and are accompanied by 4-6 Hz spike–and–slow wave and polyspike–and–slow wave discharges. These are usually associated with juvenile myoclonic epilepsy.
Benign childhood epilepsy with centrotemporal spikes (BECTS) - Most common. Typically starts during childhood and is outgrown in adolescence. The child typically wakes up at night owing to a simple partial seizure causing buccal and throat tingling and tonic or clonic contractions of one side of the face, with drooling and inability to speak but with preserved consciousness and comprehension. Complex partial and secondary generalized seizures can also occur. EEG shows typical broad-based centrotemporal spikes that are markedly increased in frequency during drowsiness and sleep. MRI is normal. Patients respond very well to AEDs such as carbamazepine. In some patients who only have rare and mild seizures treatment might not be needed.

Nocturnal autosomal dominant frontal lobe epilepsy has been linked to acetylcholine-receptor gene mutations and manifests with nocturnal seizures with dystonic posturing that respond promptly to carbamazepine. Several other less-frequent familial benign epilepsy syndromes with different localizations have also been described, some of which occur exclusively or predominantly in adults.

Parasomnias are defined as episodic nocturnal behaviors that often involve cognitive disorientation and autonomic and skeletal muscle disturbance. Parasomnias may be further characterized as occurring primarily during NREM sleep (partial arousal parasomnias) or in association with REM sleep, including nightmares, hypnogogic hallucinations, and sleep paralysis; other common parasomnias include sleep-talking.
Temporal Lobe Epilepsy - Activation of temporal discharges in sleep can lead to loss of speech and verbal auditory agnosia (Landau-Kleffner epileptic aphasia syndrome).

How well did you know this?
1
Not at all
2
3
4
5
Perfectly
21
Q

A seven-year-old child with delayed language development, aggressive behaviour and seizures is brought to see you for evaluation. His mother is concerned about the increasing size of a “birth mark” on his abdomen which is shown below.

What other findings are you most likely to find on clinical examination?

A. Adenoma sebaceum.
B. Axillary freckling.
C. Café au lait spots.
D. Shagreen patch.
E. Ungual fibroma

A

A. Adenoma sebaceum.
The image shows a hypopigmented lesion seen in Tuberous Sclerosis Complex. Adenoma sebaceum are angiofibromas which develop over nose and cheeks at 4-6 years of age and are often confused with acne. A shagreen patch is also characteristic of TSC and consists of a roughened, raised lesion with an orange-peel consistency located primarily in the lumbosacral region. During adolescence or later, small fibromas or nodules of skin may form around fingernails or toenails in 15-20% of the TSC patients.

Cafe au lait and axillary freckling are skin lesions in neurofibromatosis.

How well did you know this?
1
Not at all
2
3
4
5
Perfectly
22
Q

Cerebral palsy is defined as a disorder of posture and movement due to an insult to the developing brain. What is the most common period for the responsible brain insult to occur?
A. Antenatal.
B. Intrapartum.
C. Neonatal.
D. 1 to 12 months.
E. After 12 months.

A

A. Antenatal.

Prematurity (78 percent)
Intrauterine growth restriction (34 percent)
Intrauterine infection (28 percent)
Antepartum hemorrhage (27 percent)
Severe placental pathology (21 percent)
Multiple pregnancy (20 percent)

How well did you know this?
1
Not at all
2
3
4
5
Perfectly
22
Q

An 11-year-old child undergoes muscle biopsy under general anaesthetic for a suspected myopathy. During the procedure she develops a high temperature and generalised stiffness. Biochemical testing shows acidosis, hypercapnia and a markedly elevated serum creatine kinase (CK).
The most likely diagnosis is:
A. halothane allergy.
B. malignant hyperthermia.
C. mitochondrial encephalomyopathy with lactic acidosis and stroke like episodes (MELAS).
D. neuroleptic malignant syndrome.
E. sepsis.

A

B. Malignant hyperthermia

**_Malignant hyperthermia_**
 Malignant hyperthermia (MH) is a genetic disorder of skeletal muscle metabolism that can manifest clinically as a hypermetabolic crisis in genetically predisposed individuals who are exposed to inhalational anesthetics or depolarizing muscle relaxants. Rarely, susceptible individuals may trigger a reaction when exposed to extremely hot conditions or during vigorous exercise.

The diagnosis of acute MH is based upon clinical signs, the most reliable of which is hypercapnia that is due to a mixed metabolic and respiratory acidosis and is resistant to increasing the patient’s minute ventilation. Other early clinical signs include sinus tachycardia and muscle rigidity. Hyperthermia is a later sign of MH and may be absent when the diagnosis is initially suspected.

Neuroleptic malignant syndrome
Neuroleptic malignant syndrome (NMS) is a life threatening neurologic emergency associated with the use of neuroleptic agents and characterized by a distinctive clinical syndrome.

The diagnosis should be suspected when any two of the four cardinal clinical features, mental status change, rigidity, fever, or dysautonomia, appear in the setting of neuroleptic use or dopamine withdrawal.

MELAS
The syndrome of mitochondrial encephalopathy with lactic acidosis and stroke-like episodes (MELAS) is a maternally inherited multisystemic disorder caused by mutations of mitochondrial DNA. The hallmark of this syndrome is the occurrence of stroke-like episodes that result in hemiparesis, hemianopia, or cortical blindness. Other common features include focal or generalized seizures, recurrent migraine-like headaches, vomiting, short stature, hearing loss, and muscle weakness. A multitude of transfer RNA mutations can be responsible for MELAS but 80 percent of cases are related to the A3243G mutation and 10 percent to the T3271C transfer RNA mutation.

MELAS usually manifests in childhood after a normal early development. A relapsing-remitting course is most common, with stroke-like episodes leading to progressive neurologic dysfunction and dementia.
The original diagnostic criteria for MELAS required stroke-like episodes before age 40 years, encephalopathy characterized by seizures or dementia, and either blood lactic acidosis or the presence of ragged red fibers in skeletal muscle biopsy. However, a broader range of phenotypes is now recognized as compatible with the diagnosis, including clinical onset after age 40.

How well did you know this?
1
Not at all
2
3
4
5
Perfectly
24
Q

A term infant with a history of neonatal sepsis was noted to have developed an area of subcutaneous
fat necrosis over the left spiral groove of the humerus. She has a left-sided wrist drop as shown in the
photograph below. Her deep tendon reflexes are normal.


What nerve is most likely affected?
A. Axillary.
B. Median.
C. Musculocutaneous.
D. Radial.
E. Ulnar.

A

D. Radial.

Axillary Nerve

  • Supplies deltoid and teres minor, and sensory information of shoulder joint

Median Nerve

  • Invervates all flexors of the forearm except flexor carpi ulnaris and part of the flexor digitorum profundus.
  • The muscles of the hand supplied by the median nerve can be remembered using the mnemomic, “LOAF” for Lumbricals 1 & 2, Opponens pollicis, Abductor pollicis brevis and Flexor pollicis brevis. (NB: OAF are the thenar eminence)

Musculocutaneous Nerve

  • In its course through the arm it innervates the coracobrachialis, biceps brachii, and the greater part of the brachialis.

Radial Nerve

  • The radial nerve descends down the medial upper arm before wrapping around the mid-humerus and taking a more posterior course.
  • The nerve is particularly predisposed to compression in the region where it runs adjacent to the humerus, known as the spiral groove. Compression of the nerve in this area often occurs after prolonged pressure on the nerve. The term “Saturday night palsy” has been applied to this disorder since inebriated individuals frequently develop the problem.
  • On examination, the triceps retains full strength but there is weakness of the wrist extensors (ie, “wrist drop”), finger extensors, and brachioradialis. Sensory loss over the dorsum of the hand, possibly extending up the posterior forearm, may also be present.

Ulnar Nerve

  • Symptoms include sensory loss and paresthesias over digits 4 and 5.
  • In more severe cases, weakness of the interosseous muscles of the hand becomes apparent and the patient may complain of worsened grip and clumsiness.
  • Pain in the region of the elbow also is common, although not universal. Involvement of ulnar innervated forearm muscles leads to weakness in finger and wrist flexion.

Subcutaneous fat necrosis

  • uncommon
  • usually occurs in the first few weeks of life as a result of ischemia to the adipose tissue following a traumatic delivery.
  • characterized by firm, indurated nodules and plaques on the back, buttocks, thighs, forearms, and cheeks. The nodules and plaques may be erythematous, flesh colored, or blue.
  • self-limiting, with resolution usually occurring by six to eight weeks of age.
  • long-term follow-up for the development of hypercalcemia, which can occur up to six months after the initial presentation of the skin lesions.
How well did you know this?
1
Not at all
2
3
4
5
Perfectly
24
Q

Phenobarbitone is commonly used as the first line anticonvulsant in the treatment of neonatal
seizures. In what percentage of cases would you expect to achieve clinical control of seizures after a loading dose of 20 mg/kg phenobarbitone?
A. 20%
B. 40%
C. 60%
D. 80%
E. 95%

A

B. 40%

How well did you know this?
1
Not at all
2
3
4
5
Perfectly
25
Q

An 11-year-old boy is seen in clinic for review of congenital cardiac disease, for which he underwent
surgery as a neonate. You notice asymmetry of his face, as demonstrated in the photograph below.


These findings are consistent with:
A. Duane syndrome.
B. Horner syndrome.
C. Mobius syndrome.
D. Waardenburg syndrome.
E. Williams syndrome.

A

B. Horner Syndrome

Horner Syndrome

  • The degree of anisocoria is more marked in the dark than in light. There is associated dilation lag, an asymmetry in pupillary redilation between the two eyes when the light source is moved away from the eye. The Horner’s pupil will redilate more slowly (by 15 to 20 seconds) than the normal pupil.
  • The ptosis is minor (less than 2 mm) and occurs as a result of paralysis of the Müller’s muscle, which is innervated by the sympathetic pathway. The lower as well as the upper lid is affected, producing the so-called “upside-down ptosis.” This further narrows the palpebral fissure. The levator palpebrae superioris is unaffected; weakness of this muscle produces the more profound upper lid ptosis seen in third cranial nerve palsies.
  • Anhidrosis is present in central or preganglionic (first or second-order) lesions. The sympathetic fibers responsible for facial sweating and vasodilation branch off at the superior cervical ganglion from the remainder of the oculosympathetic pathway; thus, anhidrosis is not a feature of postganglionic or third-order lesions. This sign is frequently not apparent to patients or clinicians.
  • In infants and children, impaired facial flushing (Harlequin sign) is often more apparent than anhidrosis. Acute features of sympathetic disruption can also include ipsilateral conjunctival injection, nasal stuffiness, and increased near point of accommodation.
  • A congenital Horner’s syndrome should be suspected when anisocoria is associated with heterochromia (unequal iris color, with the affected iris being lighter). This occurs because formation of iris pigment in the first several months of age is under sympathetic control. This may only be apparent if the natural color is relatively dark.

Duane Syndrome

  • congenital strabismus that is usually caused by failure of normal development of the abducens nerve (the sixth cranial nerve) and abnormal innervation of the lateral rectus muscle by the oculomotor nerve.
  • Patients with Duane syndrome may have esotropia or exotropia

Mobius Syndrome

  • congenital facial palsy (unilateral or bilateral) with abnormalities of abducens (cranial nerve VI) function, though other cranial nerves (III, IV, V, VIII) may be involved.

Waardenburg Syndrome

  • autosomal dominant inherited pigmentary disorder
  • abnormal distribution of melanocytes during embryogenesis results in patchy areas of depigmentation.
  • Several forms of Waardenburg syndrome are described. All have the clinical features of type 1, which is characterized by a piebald-like distribution of patchy depigmentation of the hair and skin.
  • Other distinctive noncutaneous features include pigmentary abnormalities of the iris (heterochromia irides) and a broad nasal root, secondary to lateral displacement of the inner canthi of the eyes.
  • Congenital deafness occurs in one in five patients with Waardenburg syndrome and, conversely, an estimated 2 to 35 percent of cases of congenital deafness results from the disorder.

Williams Syndrome

  • Dysmorphic facial features described as elfin or pixie-like, which consist of a broad forehead, medial eyebrow flare, strabismus, flat nasal bridge, malar flattening, a short nose with a long philtrum, full lips, and a wide mouth.
How well did you know this?
1
Not at all
2
3
4
5
Perfectly
25
Q

A 3200 g term baby girl, born by ventouse extraction, is seen at five hours of age with recurrent apnoeic episodes. On examination she is afebrile, pale but well perfused, heart rate is 150 beats/min and mean arterial pressure is 42 mmHg. A computed tomography (CT) scan of her head is shown below.

The scan is most suggestive of haemorrhage that is both intracerebral and:
A. extradural.
B. intraventricular.
C. subarachnoid.
D. subdural.
E. subgaleal.

A

C. subarachnoid.

  • Extradural - lenticular
  • Subarachnoid - follow sulci and gyri
  • Subdural - crescent
  • subgaleal - between subcutaneous tissue and periosteum
  • cephalohaematoma - between skull and periosteum
  • caput succedaneum - within subcutaneous tissue
How well did you know this?
1
Not at all
2
3
4
5
Perfectly
26
Q

A five-year-old boy is brought to the Emergency Department. His parents report that he was well when he woke, but by midday he was so unsteady he could not walk. On examination he looks well but frightened. His fundi are normal and his eye movements full, but he has nystagmus on lateral gaze. He has an unsteady gait and falls frequently, but his strength and reflexes are normal.

What is the most likely diagnosis?

A. Guillain-Barré syndrome.
B. Medulloblastoma.
C. Migraine.
D. Post-infectious cerebellitis.
E. Stroke.

A

D. Post-infectious cerebellitis.

Post Infectious Cerebellitis

  • aka Acute Cerebellar Ataxia
  • Known complication of Varicella.
  • 1/4000 cases varicella in unvaccinated children
  • gradual onset of gait disturbance, nystagmus (lateral gaze), and slurred speech.
  • Neurologic symptoms usually begin 2-6 days after the onset of the rash but may occur during the incubation period or after resolution of the rash.
  • Clinical recovery is typically rapid, occurring within 24-72 hr, and is usually complete.

Guillain-Barré syndrome

  • postinfectious polyneuropathy involving mainly motor but sometimes also sensory and autonomic nerves
  • paralysis usually follows a nonspecific viral infection by about 10 days
  • Weakness usually begins in the lower extremities and progressively involves the trunk, the upper limbs, and finally the bulbar muscles, a pattern known as Landry ascending paralysis.
  • onset is gradual and progresses over days or weeks.
  • Can cause muscle pain on palpation, paraesthesis, refusal to walk and irritability

Medulloblastoma

  • 2-3 mo of headaches, vomiting, truncal ataxia

Migraine

  • Often bilateral in children
  • Can be assoc with N&V
  • Aura
  • Sometimes with photophobia or phonophobia
  • Sometimes assoc with hemiplegia
  • Usually lasts 1-72 hours

Stroke

  • The predominant presentation of children with stroke consists of the abrupt onset of focal neurologic deficits, and that of children with intracerebral hemorrhage is coma (if large portions of the cortex or brainstem are involved)
How well did you know this?
1
Not at all
2
3
4
5
Perfectly
27
Q

An eight-and-a-half-year-old boy is referred for review of his multiple birth marks as demonstrated in the clinical photograph below. He is well, has no family history of note and has normal growth and development. Physical examination is normal apart from mild pectus excavatum and a soft ejection systolic murmur.


The test or procedure most likely to assist in confirming his suspected diagnosis is:
A. computed tomography (CT) scan of his head.
B. echocardiogram.
C. fundoscopy.
D. skeletal survey.
E. slit lamp examination.

A

E. slit lamp examination.

Neurofibromatosis

Diagnosis with 2 of following 7 criteria

  1. Six or more café-au-lait macules over 5 mm in greatest diameter in prepubertal individuals and over 15 mm in greatest diameter in postpubertal individuals.
  2. Axillary or inguinal freckling consisting of multiple hyperpigmented areas 2-3 mm in diameter. Skinfold freckling usually appears between 3 and 5 yr of age. The frequency of axillary and inguinal freckling has been reported to be greater than 80% by 6 yr of age.
  3. Two or more iris Lisch nodules. Lisch nodules are hamartomas located within the iris and are best identified by a slit-lamp examination. Present in >74% of patients with NF-1 but are not a component of NF-2. The prevalence of Lisch nodules increases with age, from only 5% of children
  4. Two or more neurofibromas or 1 plexiform neurofibroma.
  5. A distinctive osseous lesion such as sphenoid dysplasia (which may cause pulsating exophthalmos) or cortical thinning of long bones (e.g., of the tibia) with or without pseudoarthrosis.
  6. Optic gliomas are present in approximately 15% of patients with NF-1 and represent mostly low-grade astrocytomas.
28
Q

An 11-year-old girl presents with a three day history of a painful and swollen right ear. On examination
she has asymmetrical facial movements and a swollen and erythematous right pinna, as demonstrated
in the clinical photographs below.


Which of the following is the most likely explanation for her presentation?
A. Auricular cellulitis.
B. Cholesteatoma.
C. Mastoiditis.
D. Otitis externa.
E. Ramsay-Hunt syndrome.

A

E. Ramsay-Hunt syndrome.

Ramsay-Hunt Syndrome

  • Typically includes the triad of ipsilateral facial paralysis, ear pain, and vesicles in the auditory canal and auricle.
  • Taste perception, hearing (tinnitus, hyperacusis), and lacrimation are affected in selected patients.
  • Polycranial neuropathy with frequent involvement of cranial nerves V, IX, and X.
  • Vestibular disturbances (vertigo) are also frequently reported.
  • Linked to reactivation of latent VZV residing within the geniculate ganglion with subsequent spread of the inflammatory process to involve the eighth cranial nerve. This results in auditory and vestibular disorders
30
Q

You review a 15-month-old girl whose father has a diagnosis of neurofibromatosis type 1 (NF1). Her parents want to know if she is affected. If she is affected, which feature is most likely to be present?

A. Axillary freckling.
B. Lisch nodules.
C. Multiple café au lait macules.
D. Optic glioma.
E. Plexiform neurofibroma.

A

C. Multiple café au lait macules.

Neurofibromatosis

Diagnosis with 2 of following 7 criteria

  1. Six or more café-au-lait macules over 5 mm in greatest diameter in prepubertal individuals and over 15 mm in greatest diameter in postpubertal individuals. Café-au-lait spots are the hallmark of neurofibromatosis and are present in almost 100% of patients. They are present at birth but increase in size, number, and pigmentation, especially during the first few yrs of life
  2. Axillary or inguinal freckling consisting of multiple hyperpigmented areas 2-3 mm in diameter. Skinfold freckling usually appears between 3 and 5 yr of age. The frequency of axillary and inguinal freckling has been reported to be greater than 80% by 6 yr of age.
  3. Two or more iris Lisch nodules. Lisch nodules are hamartomas located within the iris and are best identified by a slit-lamp examination. Present in >74% of patients with NF-1 but are not a component of NF-2. The prevalence of Lisch nodules increases with age, from only 5% of children
  4. Two or more neurofibromas or 1 plexiform neurofibroma.
  5. A distinctive osseous lesion such as sphenoid dysplasia (which may cause pulsating exophthalmos) or cortical thinning of long bones (e.g., of the tibia) with or without pseudoarthrosis.
  6. Optic gliomas are present in approximately 15% of patients with NF-1 and represent mostly low-grade astrocytomas.
32
Q

Antipsychotic medications such as risperidone and haloperidol are used by paediatricians to treat Tourette syndrome, and the symptoms of aggression and anxiety in autism. An acute side effect of these medications is painful and distressing muscle spasms. These can be reversed by which of the following?

A. Benztropine.
B. Cyproheptadine.
C. Diazepam.
D. Flumazenil.
E. Naloxone.

A

A. Benztropine.

  • Dopamine blocking agents (antipsychotics and antiemetics) can cause drug induced movement disorders (DIMD).
  • Acute dystonic reactions, occurring in the 1st days of exposure, typically involve the face and neck, manifesting as torticollis, retrocollis, oculogyric crisis, or tongue protrusion.
  • Life-threatening presentations with laryngospasm and airway compromise can also occur, requiring prompt recognition and treatment of this entity.
  • Delayed onset involuntary movements, tardive dyskinesias, develop in the setting of chronic neuroleptic use, at least 3 mo in duration. Involvement of the face, particularly the mouth, lips, and/or jaw with chewing or tongue thrusting is characteristic.

Benztropine MOA

Possesses both anticholinergic and antihistaminic effects. May also inhibit the reuptake and storage of dopamine, thereby prolonging the action of dopamine.

34
Q

A 14-year-old girl with a seizure disorder presents because of ongoing seizures. She was initially
diagnosed with “grand mal” epilepsy following her first tonic-clonic seizure at 12 years of age.
Her teachers have expressed concern about brief periods of “tuning out” in the classroom. Her
parents tell you that she is clumsy and seems “jumpy” first thing in the morning.
Her electroencephalogram (EEG) shows generalised spike waves.
Which of the following anticonvulsants is most likely to worsen her seizures?
A. Carbamazepine.
B. Clonazepam.
C. Phenobarbitone.
D. Sodium valproate.
E. Topiramate.

A

A. Carbamazepine.

Carbamazepine and phenytoin have been reported to worsen absence and myoclonic seizures in individuals with idiopathic generalized epilepsy.

36
Q

A two-year-old girl presents with a three month history of deteriorating gait and behavioural disturbance. She was born at 33 weeks gestation with an uncomplicated neonatal course. Prior to presentation there were no concerns on the part of her parents. On examination she was ataxic, ankle jerks were absent and plantar responses upgoing. A magnetic resonance imaging (MRI) scan is performed and shown below.

What is the most likely diagnosis?
A. Adrenoleukodystrophy.
B. Ataxia telangiectasia.
C. Friedreich ataxia.
D. Metachromatic leukodystrophy.
E. Periventricular leukomalacia.

A

D. Metachromatic leukodystrophy.

Metachromatic leukodystrophy
Autosomal recessive white matter disease
Late infantile form of MLD, most common MLD.

  • usually presents between 12 and 18 mo of age as irritability, inability to walk, and hyperextension of the knee, causing genu recurvatum.
  • The clinical progression of the disease relates to the pathological involvement of both central and peripheral nervous system, giving a mixture of upper and lower motor neuron and cognitive and psychiatric signs.
  • Deep tendon reflexes are diminished or absent.
  • Gradual muscle wasting, weakness, and hypotonia become evident and lead to a debilitated state.
  • As the disease progresses, nystagmus, myoclonic seizures, optic atrophy, and quadriparesis appear, with death in the 1st decade of life

Adrenoleukodystrophy

  • Peroxisomal disorder.
  • X linked recessive so occurs only in males.
  • Boys present between 5 and 15 yr of age with evidence of academic difficulties, behavioral disturbances, and gait abnormalities.
  • It is a white matter demyelination involving occipital lobes and splenium in bilateral and symmetric pattern (demyelination moves from centre to periphery).

Ataxia Telangiectasia

  • Progressive cerebellar ataxia, abnormal eye movements, other neurologic abnormalities, oculocutaneous telangiectasias, and immune deficiency.
  • MRI - cerebellar atrophy

Friedreich Ataxia

  • Autosomal recessive disorder involving the spinocerebellar tracts, dorsal columns in the spinal cord, the pyramidal tracts, and the cerebellum and medulla.
  • Ataxia usually before age 10 years
  • Progressive staggering gait, frequent falling, and titubation.
  • Lower extremities are more severely involved than the upper ones.
  • Dysarthria occasionally is the presenting symptom; rarely, progressive scoliosis, foot deformity, nystagmus, or cardiopathy is the initial sign.
  • The neurologic examination reveals nystagmus, loss of fast saccadic eye movements, truncal titubation, dysarthria, dysmetria, and ataxia of trunk and limb movements. Extensor plantar responses (with normal tone in trunk and extremities), absence of deep tendon reflexes, and weakness (greater distally than proximally) are usually found. Loss of vibratory and proprioceptive sensation occurs. The median age of death is 35 years. Women have a significantly better prognosis than men.

Periventricular Leukomalacia

  • White-matter brain injury secondary to IVH in preterm infants.
  • Necrosis around lateral ventricles.
  • PVL is usually clinically asymptomatic until the neurologic sequelae of white matter damage become apparent in later infancy as spastic motor deficits.
  • PVL may be present at birth but usually occurs later as an early echodense phase (3-10 days of life), followed by the typical echolucent (cystic) phase (14-20 days of life).
38
Q

You are asked to see a four-month-old in the Intensive Care Unit. He has been hypotonic and weak since birth. On examination, there is no facial weakness but he has profound weakness of the extremities with areflexia. His chest X-ray is shown.


The most likely diagnosis is:
A. congenital myotonic dystrophy.
B. mitochondrial myopathy.
C. nemaline myopathy.
D. Pompe disease.
E. spinal muscular atrophy.

A

D. Pompe Disease

Pompe Disease

  • Present in the 1st few months of life with hypotonia, a generalized muscle weakness with a “floppy infant” appearance, neuropathic bulbar weakness, feeding difficulties, macroglossia, hepatomegaly, and a hypertrophic cardiomyopathy followed by death from cardiorespiratory failure or respiratory infection usually by 1 yr of age.
  • Heart usually massive on CXR

Congenital Myotonic Dystrophy

  • Neuromuscular disease
  • Characterized by profound hypotonia, facial diplegia, poor feeding, arthrogryposis (especially of the legs), and respiratory failure [10]. Affected infants have a characteristic “V” shape of the upper lip that results from facial diplegia.

Mitonchondrial Myopathy

  • The clinical expression of mitochondrial myopathies is extremely variable. Myopathy may be the sole or main sign, or merely an incidental finding associated with a multisystemic illness. The severity of these disorders ranges from asymptomatic mild proximal limb weakness to fatal infantile myopathy.

Nemaline Myopathy

  • In the infantile form, generalized hypotonia and weakness, which can include bulbar-innervated and respiratory muscles, and a very thin muscle mass are characteristic.
  • The head is dolichocephalic, and the palate high arched or even cleft. Muscles of the jaw may be too weak to hold it closed. Decreased fetal movements are reported by the mother, and neonates suffer from hypoxia and dysphagia; arthrogryposis may be present.

Spinal Muscular Atrophy

  • The cardinal features of SMA type 1 are severe hypotonia; generalized weakness; thin muscle mass; absent tendon stretch reflexes; involvement of the tongue, face, and jaw muscles; and sparing of extraocular muscles and sphincters. Diaphragmatic involvement is late. Infants who are symptomatic at birth can have respiratory distress and are unable to feed.
  • Congenital contractures, ranging from simple clubfoot to generalized arthrogryposis, occur in about 10% of severely involved neonates.
  • Infants lie flaccid with little movement, unable to overcome gravity. They lack head control. More than 65% of children die by 2 yr of age, and many die early in infancy.
40
Q

In mild diffuse acquired brain injury in childhood which of the following is most likely to be impaired?
A. Executive function.
B. Hearing.
C. Motor control.
D. Speech.
E. Vision.

A

A. Executive function.

42
Q

A six-year-old girl is seen in the Emergency Department after falling from a swing and hitting her head. The history suggests loss of consciousness at the scene. On examination her Glasgow Coma Score is 14/15. Head circumference is 47.5 cm (

This most likely diagnosis is:

A. craniosynostosis.
B. hydrocephalus.
C. Langerhans cell histiocytosis.
D. osteopetrosis.
E. periostotic fibrous dysplasia.

A

A. Craniosynostosis. Early closure of sutures limiting brain growth.

Hydrocephalus would cause large head.
Langerhans cell histiocytosis - lytic skull lesions
Osteopetrosis - marble bone disease - bright bone on xray. Osteoclastic dysfunction.
Periostotic fibrous dysplasia - fibrous growths on bone

43
Q

A six-month-old girl is referred to the outpatient clinic for assessment of delayed development. She will not reach, sit unsupported or roll. She weighs 7.6 kg (50th - 75th percentile), and measures 67cm in length (50th percentile). Her head circumference is 41 cm (25th percentile). Her head and facial appearance is shown below.

The most likely reason for this appearance is:
A. coronal synostosis.
B. deformational plagiocephaly.
C. lambdoid synostosis.
D. metopic synostosis.
E. sagittal synostosis.

A

D. metopic synostosis.

These children have a keel-shaped forehead and hypotelorism and are at risk for associated developmental abnormalities of the forebrain. Milder forms of metopic ridging are more common.

44
Q

A 13-year-old girl presents with intermittent headaches for the last two years. They have become more frequent in recent months and now occur most mornings. A magnetic resonance imaging (MRI) scan is performed and shown below.
 
The most likely diagnosis is:

A. brainstem glioma.
B. craniopharyngioma.
C. ependymoma.
D. medulloblastoma.
E. pilocytic astrocytoma.

A

E. pilocytic astrocytoma.

Pilocytic Astrocytoma

  • Astrocytomas account for 40% of childhood CNS tumours, PA 20% of all brain tumours
  • PA can occur anywhere in CNS, classic site is cerebellum
  • Classic radiological finding in PA is the presence of a contrast medium–enhancing nodule within the wall of a cystic mass (not exclusive to PA)
  • Indolent tumour with low metastatic potential and is rarely invasive.

Brainstem Glioma

  • Diffuse pontine most common.
  • Present with motor weakness, cranial nerve deficits, cerebellar deficits, and/or signs of increased ICP
  • Terrible prognosis, median survival 12mo
  • MRI usually homogenous pre-treatment, necrosis present in some pts
  • T1 - decreased intensity
    T2 - heterogeneously increased

Craniopharyngioma

  • Grade I tumour
  • Common - 7-10% of all childhood tumours
  • often present with endocrinologic abnormalities such as growth failure and delayed sexual maturation. Visual changes can occur and may include decrease acuity or visual field deficits.
  • MRI demonstrates the solid tumor with cystic structures containing fluid of intermediate density in suprasellar region.

Ependymoma

  • Accounts for 10% childhood tumours
  • Arises from ependymal lining of ventricles.
  • 70% occur in posterior fossa
  • Mean age 6 yrs
  • MRI demonstrates a well-circumscribed tumor with variable and complex patterns of gadolinium enhancement, with or without cystic structures
  • Clinical presentation can be insidious and often depends on the anatomic location of the tumor

Medulloblastoma

  • Embryonal tumour of cerebellum occuring almost exclusively in males 5-7 years.
  • CT and MRI demonstrate a solid, homogeneous, contrast medium–enhancing mass in the posterior fossa causing 4th ventricular obstruction and hydrocephalus.
  • Patients present with signs and symptoms of increased ICP (i.e., headache, nausea, vomiting, mental status changes, and hypertension) and cerebellar dysfunction (i.e., ataxia, poor balance, dysmetria).
45
Q

A 13-year-old boy collapses while playing soccer. There is a brief period of loss of consciousness, but he is now awake, alert and oriented, although complaining of a severe headache. A computed tomography (CT) scan of his head is shown below.


Which of the following is the most likely
diagnosis?
A. Diffuse cerebral oedema.
B. Extradural haemorrhage.
C. Migraine.
D. Subarachnoid haemorrhage.
E. Subdural haemorrhage

A

D. Subarachnoid haemorrhage.

46
Q

A 5 month old boy with congenital heart disease is evaluated because of motor delay.
A computed tomography (CT) scan of his brain is performed and is shown below.

What is the most likely diagnosis?
A. Arachnoid cyst.
B. Cerebral malformation.
C. Extradural haematoma.
D. Porencephalic cyst.
E Subdural effusion.

A

Arachnoid Cyst
CT scans:

  • characterized by sharp, nonenhancing borders
  • isodense to CSF
  • remodeling of the skull may be evident on a bone window
  • seldom calcify
47
Q

What is the most common side-effect of clonidine?
A. Dizziness.
B. Dry mouth.
C. Rash.
D. Sedation.
E. Tics.

A

B. Dry mouth.

48
Q

An obese 14-year-old girl is commencing antiepileptic medication. Which anticonvulsant is most likely to cause further weight gain?

A. Carbamazepine.

B. Clobazam.

C. Lamotrigine.

D. Phenytoin.

E. Sodium valproate.

A

E. Sodium Valproate

49
Q

A 15-year-old obese girl complains of recurrent numbness in her right hand at night. On further questioning, the numbness is confined to the 4th and 5th fingers. Examination of her hands is normal except for mild sensory abnormalities of the 5th finger of the right hand.
The nerve most likely affected is:

A. axillary.
B. median.
C. musculocutaneous.
D. radial.
E. ulnar.

A

B. Median

50
Q

A healthy three-year-old girl presents with a squint. On examination, visual acuity is diminished in the left eye. The magnetic resonance imaging (MRI) scan of her head is shown opposite. Which one of the following inherited conditions is she most likely to have?

A. Familial retinoblastoma.

B. Neurofibromatosis type 1.

C. Sturge-Weber syndrome.

D. Tuberous sclerosis.

E. Von Hippel-Lindau disease.

A

B. Neurofibromatosis type 1.

MRI shows optic nerve glioma.
The diagnosis is best made by magnetic resonance imaging (MRI), which allows visualization of the entire course of the optic nerve.
Three typical patterns are seen on neuroimaging studies:
Tubular thickening of the optic nerve and chiasm
Suprasellar tumor with contiguous optic nerve expansion
Suprasellar tumor with optic tract involvement

Optic pathway gliomas occur in 15 percent of children younger than six years of age with NF1. They rarely occur in older children and adults. OPGs are typically low-grade gliomas. They can arise anywhere along the anterior visual pathway to the optic radiations and involve the optic nerves, chiasm, and postchiasmal optic tracts.

51
Q

A 14-year-girl presents with her first generalised seizure at 6 a.m. On direct questioning she has a 12- month history of limb jerks, usually in the mornings, that often result in her breakfast being spilt. Her examination is entirely normal.
Her electroencephalogram (EEG) is shown below.

A diagnosis of juvenile myoclonic epilepsy is made.

Which of the following anticonvulsants should not be prescribed?

A. Carbamazepine.
B. Clobazam.
C. Clonazepam.
D. Lamotrigine.
E. Topiramate.

A

A. Carbarmazepine

52
Q

An eight-year-old boy presents with a low-grade fever, irritability and unsteadiness. On further questioning there is a seven-week history of double vision and ataxia.
Cranial nerve examination shows a left abducens nerve palsy and mild right facial weakness. He has very brisk reflexes but no other pertinent findings. The computerised tomography (CT) scan of his brain is shown below.

The CT is most consistent with:

A. brain abscess.

B. cerebellitis.

C. disseminated encephalomyelitis.

D. glioma.

E. stroke.

A

D. Glioma
MRI shows focal pontine glioma.
Small focal tumors of the midbrain or medulla usually present insidiously, with a long history of localizing findings such as an isolated cranial nerve deficit or contralateral hemiparesis. Signs and symptoms of raised intracranial pressure are uncommon. May have pressure effect on optic nerve causing diplopia.

53
Q

A nine-month-old girl presents to the emergency department with clusters of brief jerking movements which are increasing. She has eight to ten pale patches over her trunk and limbs but is otherwise thriving and developmentally normal. A magnetic resonance imaging (MRI) scan of her brain is shown opposite.
What is the most likely diagnosis?

A. Hypomelanosis of Ito.

B. Incontinentia pigmenti.

C. Neurofibromatosis.

D. Sturge-Weber syndrome.

E. Tuberous sclerosis.

A

E. Tuberous sclerosis
MRI shows glioneuronal hamartomas (aka cortical tubers) and subependymal nodules. Other MRI findings in TSC include subependymal giant cell tumours, and white matter abnormalities. Hypomelanotic lesions are called ash leaf spots, which is one of the most common clinical features, as well as shagreen patch, angiofibromas/fibroadenomas, and brown fibrous plaque on forehead. Seizures occur in 80-90% pts and 60% develop by age 1 year. Infantile spasm most common type of seizure.

Sturge-Weber syndrome is a vascular disorder with features including port wine stain, leptomeningeal angioma. Often present with seizures, stroke-like episodes, hemiparesis, headaches and developmental delay. Incontinentia pigmenti is a rare x-linked dominant mosaic disorder which is lethal in males and features dermatologic, dental and occular abnormalities. Four phases of the disease with Phase 3 the hallmark of incontinentia pigmenti and involves hyperpigmentation in macular whorls that follow Blashko lines. (Progression from eosinophilic vesicles and verrucous plaques). In stage 4 lesions become hypopigmented and hairless. Hypomelanosis of Ito is a congenital skin disorder with associated abnormalities including mental retardation, seizure, microcephaly and hypotonia. The skin lesions of hypomelanosis of Ito are generally present at birth but may be acquired in the first 2 years of life. The lesions are similar to a negative image of those present in incontinentia pigmenti, consisting of bizarre, patterned, hypopigmented macules arranged over the body surface in sharply demarcated whorls, streaks, and patches that follow the lines of Blaschko.

54
Q

A five-year-old girl presents with a history of regression of language and motor skills. Examination shows deficits in cognition, language, and motor development. She has bilateral lower limb hypertonia with mild distal weakness, absent deep tendon reflexes and extensor plantar responses. She is very ataxic when walking.
Magnetic resonance imaging (MRI) of her brain is shown below.

Which of the following is the most likely diagnosis?

A. Adrenoleukodystrophy.

B. Krabbe disease.

C. Leigh disease.

D. Metachromatic leukodystrophy.

E. Tuberous sclerosis.

A

D. Metachromatic leukodystrophy

Adrenoleukodystrophy - X linked recessive, usually seen in males, milder form in female carriers after age 35 years. Boys typically present with learning disabilities and behavior problems that are often diagnosed as attention deficit hyperactivity disorder, and may respond to stimulant medication. This is followed by neurologic deterioration that includes increasing cognitive and behavioral abnormalities, blindness, and the development of quadriparesisPosterior dominance of lesion on MRI. ALD presents between four and eight years of age (peak seven years). It is rare after 15 years of age and almost never occurs before age three.

MRI shows occipitoparietal predominance of white matter demyelination.

Metachromatic leukodystrophy rare autosomal recessive lysosomal storage disease. 3 subtypes - late infantile, juvenile and adult. Late infantile 6m-2yrs present with regression of motor skills, gait difficulties, seizures, ataxia, hypotonia, extensor plantar responses, and optic atrophy. Deep tendon reflexes are sometimes reduced or absent, reflecting a peripheral neuropathy. The prognosis is worse than later onset forms of MLD; progression to death typically occurs within five to six years. Juvenile - some children present at 4-6 years with intellectual impairment, behavioral difficulties, gait disturbance, ataxia, upper motor neuron signs, and a peripheral neuropathy. Seizures may occur, and progression to death occurs within six years of onset. Another group of children present between 6 and 16 years of age (late juvenile) with behavioral changes and intellectual impairment and, in many cases, seizures. Progression is slower, and these children may survive until early adulthood.

Brain MRI reveals symmetric white matter lesions with a periventricular predominance in the early form of the disease and cortical atrophy in the later forms.

Krabbe Disease is a rare autosomal recessive lysosomal storage disorder. Infantile, juvenile and adult onset. 90% pts present by age 6 months. Peripheral motor neuropathy occurs in all patients.

Infantile onset — Symptoms usually develop from ages two to five months with infantile onset Krabbe disease. Manifestations include irritability, developmental delay or regression, limb spasticity, axial hypotonia, absent reflexes, optic atrophy, and microcephaly. Seizures eventually appear, and tonic extensor spasms occur upon stimulation with light, sound, or touch. These children regress rapidly to a decerebrate condition, with most dying before reaching two years of age.

Juvenile onset — Patients with juvenile onset disease typically present with weakness, loss of skills, and vision loss. Late infantile and juvenile patients regress at an unpredictable rate, but all become severely incapacitated and die two to seven years after diagnosis.

Brain MRI typically (but not always) shows generalized atrophy with increased T2 signal from the periventricular and deep cerebral white matter, predominantly involving the parieto-occipital lobes. Cerebellar white matter and deep gray matter involvement are often present in early onset disease

Leigh Disease is a subacute necrotising encephalomyopathy. In decreasing order of severity, the affected areas are the basal ganglia, brainstem cerebellum, and cerebral cortex .

Classic presentation:
infant who presents with central hypotonia, developmental regression or arrest, and signs of brainstem or basal ganglia involvement.

Diagnosis is usually confirmed by radiologic or pathologic evidence of symmetric lesions affecting the basal ganglia, brainstem, and subthalamic nuclei.
Patients with Leigh disease frequently present with developmental delay, seizures, altered consciousness, failure to thrive, pericardial effusion, and dilated cardiomyopathy.

The prognosis for Leigh syndrome is poor. In a study of 14 cases, there were 7 fatalities before the age of 1.5 yr.

Brain MRI shows abnormal white matter signal in the putamen, basal ganglia, and brainstem on T2 and FLAIR sequence images.

Tuberous Sclerosis - MRI findings in TSC include glioneuronal hamartomas (aka cortical tubers), subependymal nodules, subependymal giant cell tumours, and white matter abnormalities. Hypomelanotic lesions are called ash leaf spots, which is one of the most common clinical features, as well as shagreen patch, angiofibromas/fibroadenomas, and brown fibrous plaque on forehead. Seizures occur in 80-90% pts and 60% develop by age 1 year. Infantile spasm most common type of seizure.

55
Q

A 15-year-old girl presents with a four-day history of increasing difficulty with vision. She initially noticed some pain around her right eye, then difficulty reading. Her symptoms progressed fairly rapidly over the next 48 hours to the point where she can only count fingers out of her right eye.

Examination reveals a right afferent pupillary defect, mild blurring of the right disk margin, and
severely impaired vision. Her left eye is normal, as is the rest of her neurological examination.

Past history is significant for an episode of tinnitus and vertigo, which lasted for about one week one year ago, and an episode of numbness of her left arm lasting two weeks six months ago. Both these episodes resolved spontaneously.

A magnetic resonance imaging (MRI) scan of her brain is performed and is shown below.

Which one of the following is the most likely diagnosis?

A. Acute disseminated encephalomyelitis.

B. Multiple sclerosis.

C. Subacute sclerosing panencephalitis.

D. Systemic lupus erythematosus (SLE).

E. Toxoplasmosis.

A

B. Multiple Sclerosis

Multiple Sclerosis is a chronic demyelinating disorder of the brain, spinal cord and optic nerves characterized by a relapsing-remitting course of neurologic episodes separated in time and space. Only 2-5% of pts have first symptoms before age 18 years.

A complex interplay of environmental, infectious, and genetic factors influence MS susceptibility. Immune system dysregulation involving T and B lymphocytes triggers inflammation, axonal demyelination, axonal loss, and regeneration within both white and gray matter. Inflammatory infiltrates within actively demyelinating lesions of relapsing-remitting MS are targets for disease modifying therapy (DMT). Neurodegenerative changes predominate in progressive forms of MS.

Presenting symptoms in pediatric MS include hemiparesis or paraparesis, unilateral or bilateral optic neuritis, focal sensory loss, ataxia, diplopia, dysarthria, or bowel/bladder dysfunction.

Cranial MRI exhibits discrete T2 lesions in cerebral white matter, particularly periventricular regions as well as brainstem, cerebellum, and juxtacortical and deep gray matter.

ADEM is an initial inflammatory, demyelinating event with multifocal neurologic deficits, typically accompanied by encephalopathy.

Initial symptoms of ADEM may include lethargy, fever, headache, vomiting, meningeal signs, and seizure, including status epilepticus. Encephalopathy is a hallmark of ADEM, ranging from ongoing confusion to persistent irritability to coma. Focal neurologic deficits can be difficult to ascertain in the obtunded or very young child but common neurologic signs in ADEM include visual loss, cranial neuropathies, ataxia, motor and sensory deficits, plus bladder/bowel dysfunction with concurrent spinal cord demyelination.

Imaging - Head CT may be normal or show hypodense regions. Cranial MRI, the imaging study of choice, typically exhibits large, multifocal and sometimes confluent or tumefactive T2 lesions with variable enhancement within white and often gray matter of the cerebral hemispheres, cerebellum, and brainstem.

Subacute sclerosing panencephalitis - rare, progressive neurologic disorder caused by persistent measles virus infection of the CNS. Occurs in individuals who have been exposed to natural measles virus in early childhood.
Early Clinical manifestations:

  • Early in disease - personality changes, aggressive behavior, and impaired cognitive function
  • Myoclonic seizures

Later Clinical Manifestations:

  • Generalized tonic-clonic convulsions
  • hypertonia
  • choreoathetosis,
  • followed by progressive bulbar palsy, hyperthermia, and decerebrate postures.
  • papilledema in approximately 20% of the cases.
  • Optic atrophy, chorioretinitis, and macular pigmentation are observed in most patients.

The diagnosis by the typical clinical course and 1 of the following: (1) measles antibody detected in the CSF, (2) a characteristic electroencephalogram consisting of bursts of high-voltage slow waves interspersed with a normal background that occur with a constant periodicity in the early stages of the disease, and (3) typical histologic findings in the brain biopsy or postmortem specimen.
Treatment with a series of antiviral agents has been attempted without success.

Death occurs usually within 1-2 yr from the onset of symptoms.

Systemic lupus erythematosus (SLE) is a chronic autoimmune disease characterized by multisystem inflammation and the presence of circulating autoantibodies directed against self-antigens.
Most Common Clinical Manifestations:

  • fever
  • fatigue
  • hematologic abnormalities
  • arthralgia
  • arthritis
  • Renal disease in SLE is often asymptomatic (BP monitoring and urinalysis critical)
  • SLE is often characterized by periods of flare and disease quiescence or may follow a more smoldering disease course.

Toxoplasmosis -Toxoplasma gondii, an obligate intracellular protozoan, is acquired perorally, transplacentally, or, rarely, parenterally in laboratory accidents; by transfusion; or from a transplanted organ. In immunologically normal children, acute acquired infection may be asymptomatic, cause lymphadenopathy, or affect almost any organ. Once acquired, latent encysted organisms persist in the host throughout life. In immunocompromised infants or children, either initial acquisition or recrudescence of latent organisms often causes signs or symptoms related to the central nervous system (CNS).

If untreated, congenital infection often causes disease either perinatally or later in life, most frequently chorioretinitis and CNS lesions. Other manifestations such as intrauterine growth retardation, cognitive and motor deficits, fever, lymphadenopathy, rash, hearing loss, pneumonitis, hepatitis, and thrombocytopenia also occur. Congenital toxoplasmosis in infants with HIV infection may be fulminant.

56
Q

A 13-year-old girl presents to your office with a six-month history of daily headaches. The headaches are present on waking and on occasions are associated with vomiting without nausea. She has had intermittent double vision and transient episodes of loss of vision lasting seconds only. She is taking tetracyclines for acne.
On examination, she is moderately obese, afebrile, and has no neurocutaneous stigmata. She has a moderate amount of facial acne. Her blood pressure is 105/65 mmHg. Her extraocular movements show a partial left sixth nerve palsy and she has gross papilloedema. There are no other abnormal findings.
A T2-weighted magnetic resonance imaging (MRI) scan of her brain is shown below.

The most likely diagnosis is:
A. aqueductal stenosis.
B. benign intracranial hypertension (BIH).
C. craniopharyngioma.
D. meningioma.

A

A. Aqueductal stenosis

From history, looks like BIH with consistent RFs of obesity, tetracycline use. Diagnosis made on MRI which shows ventricles are dilated and filled with CSF (white) but CSF is not continuing down spinal cord due to obstruction (stenosis). Aqueductal stenosis causes symtoms of raised intracranial pressure and can be congenital, acquired (post infection, post haemorrhage), idiopathic acquired, tumour (pineal most common).

Benign Intracranial Hypertension (nka idiopathic IH, aka pseudotumour cerebri)

  • Headache is most common symptom
  • Vomiting
  • Transient visual obscurity and diplopia
  • Papilloedema always present in children with closed fontanelle
  • May have enlarged blind spot.
  • MRI shows flattening of the bulge of the optic nerve head.

Craniopharyngioma - slow growing. Pts usually symptomatic one year before diagnosis.
Symptoms:
* Visual deficits
* Endocrine abnormalities - (growth hormone deficiency most common presentation in children, delayed sexual maturation)
* Headache
* Generalised symptoms (depression, nausea, vomiting, lethargy)

Meningioma - frequent brain tumour in children. Usually benign and often asymptomatic. Can arise anywhere in dura (10% in spine). Symptoms depend on aria that lesion arises. i.e. visual defects, hearing loss, mental status changes, extremity weakness.

Temporal Lobe Tumour

Brain tumour symptoms:

  • Headaches
  • Seizures
  • N & V
  • Depressed LOC
  • Neurocognitive dysfunction
  • Weakness
  • Sensory loss
  • Aphasia
  • Visual spatial dysfunction.
57
Q

A 10-year-old boy with cyanotic congenital heart disease presents with a two-week history of throbbing bifrontal headache, worse in the morning, associated with low-grade fever.
Investigations are as follows:
haemoglobin 201 g/L [110-150]
white cell count 10.4 x 109/L [4.0-11.0]
platelet count 250 x 109/L [150-400]
erythrocyte sedimentation rate (ESR) 5 mm/hr [0-20]
serum C-reactive protein 6 mg/L [<8]
An image from a non-contrast computed tomography (CT) scan of his brain is shown below.

Which of the following is the most likely diagnosis?
A. Cerebral abscess.
B. Congenital cystic lesion.
C. Embolic stroke.
D. Intracerebral haemorrhage.
E. Tumour.

A

A. Cerebral abscess

Cerebral Abscess
Most common symptom in headache. The pain is usually localized to the side of the abscess, and its onset can be gradual or sudden. The pain tends to be severe and not relieved by aspirin or other over-the-counter pain medications. In patients with cyanotic heart disease and headache, brain abscess must always be excluded. Fever not reliable, occurs in 50%. Neck stiffness in 15%. Signs of raised intracranial pressure.

MRI better than CT. CT shows ring enhancing lesions with surrounding oedema.

Congenital Cystic Lesion
Usually asymptomatic. May cause symptoms of brain compression, raised ICP.

Embolic Stroke
The predominant presentation of children with stroke consists of the abrupt onset of focal neurologic deficits, and that of children with intracerebral hemorrhage is coma (if large portions of the cortex or brainstem are involved).

Intracerebral Haemorrhage
Patients typically present with an acute onset of a focal neurologic deficit that corresponds to the part of the brain affected.

With large hemorrhages, there is elevated intracranial pressure and suppressed level of consciousness. 

Seizures may complicate 5 to 30 percent of hemorrhages, particularly if the hematoma is more superficial than deep.

Tumour
Signs of raised ICP.

58
Q

A five-year-old girl is brought to her paediatrician because of recurrent headaches. During the physical examination a number of skin lesions were found. These are demonstrated in the photograph below.

Which of the following is the least likely to be associated with her condition?
A. Acoustic neuroma.
B. Learning difficulties.
C. Macrocephaly.
D. Pseudoarthrosis.
E. Scoliosis.

A

A. Acoustic Neuroma

Pt has NF1. Acoustic neuroma is a feature of NF2 (aka vestibular schwannoma), common feature. Benign but can cause significant morbidity. Not common finding in NF1.

59
Q

A 10-year-old girl with a longstanding history of migraine, on no medications, presents with severe headache and left upper limb weakness.
Which of the following medications should not be used at this point?
A. Codeine.
B. Cyproheptadine.
C. Dihydroergotamine.
D. Ibuprofen.
E. Propranolol.

A

D. Sodium Valproate
History is suggestive of absence seizure in which the typical history is onset 5-7 years of age, up to hundreds of events per day lasting a few seconds and accompanied by flutter or upward rolling of eyes. May have simple automatisms such as lip smacking, clothing picking, minimal falling forward of the head. No postictal period and the patient usually resumes what they were doing immediately prior to seizure.
Carbamazepine reported to worsen absence seizures. Recommended treatment varies amongst organisations. Ethosuxamide, Valproic Acid and Lamotragine are the three variations.

60
Q

A six-year-old girl presents with a six-month history of staring spells associated with eye flickering and lip smacking movements. An electroencephalogram (EEG) is performed and is shown below.

Which of the following anticonvulsants is most appropriate?
A. Carbamazepine.
B. Clonazepam.
C. Phenytoin.
D. Sodium valproate.
E. Vigabatrin.

A
61
Q

A 15-year-old boy sustains a right shoulder dislocation during a football game. The dislocation is reduced in the emergency room within hours. On review two weeks after his injury, he complains of numbness over the radial aspect of the forearm.
On examination he has sensory impairment over the lateral (radial) aspect of the forearm from just below the elbow to just above the wrist. He also has weakness of elbow flexion. The biceps deep tendon reflex is depressed. The rest of his examination is normal.
The most likely site of injury is to the:
A. axillary nerve.
B. median nerve.
C. musculocutaneous nerve.
D. radial nerve.
E. ulnar nerve.

A

C. Musculocutaneous nerve

62
Q

An otherwise normal two-year-old boy is brought to see you because of recurrent generalised febrile convulsions. He had his first febrile convulsion at 14 months of age and has had one further febrile seizure. The seizures both lasted less than one minute.
There is no family history of febrile convulsions or epilepsy.
Which of the following is the best estimate for the risk of later epilepsy?
A. 1%.
B. 2.5%.
C. 5%.
D. 10%.
E. 20%.

A

B. 2.5%.

Risk of normal population is 2.5%, this is a normal child.

63
Q

The word ‘complex’ in complex partial seizures refers to:

A. a seizure lasting >10 minutes.
B. automatisms occurring during the seizure.
C. focal motor seizures.
D. impaired consciousness during the seizure.
E. multiple seizures in a 24-hour period

A

D. impaired consciousness during the seizure.

64
Q

An eight-year-old child presents to a regional emergency department with a one-week history of weakness in both legs, stumbling gait and complaints of numbness. The family is currently under considerable stress with the suicide of a close relative and parental fighting. Neurological examination reveals apparent reduction in power, variable sensory loss and normal deep tendon reflexes. The gait is erratic.
Which of the following is the most likely diagnosis?
A. Conversion disorder.
B. Guillain-Barré syndrome.
C. Multiple sclerosis.
D. Transverse myelitis.
E. Viral myositis.

A

A. Conversion disorder.

65
Q

A seven-year-old boy wakes in the morning and cries out to his parents to ‘come quickly’. He complains to his parents that the ‘room is spinning’. The symptoms seem to be aggravated by head motion and change of position from lying down. He becomes pale, sweaty and nauseated whenever he moves or tries to sit up in bed. He is helped to the bathroom because he has difficulty standing and walking unsupported and vomits repeatedly. His parents bring him to the emergency department soon afterwards.
On examination, he is found to be perfectly conscious and cooperative if left to lie still in bed. He has left-beating nystagmus in all positions of gaze but otherwise no focal neurologic findings. His hearing is normal. He has a mild upper respiratory tract infection.
The most likely diagnosis is:

A. acute disseminated encephalomyelitis (ADEM).
B. acute labyrinthitis.
C. acute vestibular neuritis.
D. cerebellar ataxia.
E. Menière’s disease.

A

C. Acute vestibular neuritis

Meniere disease - Affected patients present with spontaneous episodic vertigo lasting for minutes to hours, usually associated with unilateral tinnitus, hearing loss, and ear fullness. The vertigo associated with Meniere disease is often severe and associated with nausea and vomiting and disabling imbalance. The disequilibrium may last for several days. Horizontal-torsional nystagmus is typically seen on examination during an attack.

Vestibular neuritis is characterized by the rapid onset of severe, persistent vertigo, nausea, vomiting, and gait instability. Physical examination findings are consistent with an acute peripheral vestibular imbalance: spontaneous vestibular nystagmus, a positive head thrust test, and gait instability without a loss of the ability to ambulate. In pure vestibular neuritis, auditory function is preserved; when this syndrome is combined with unilateral hearing loss, it is called labyrinthitis.

Acute cerebellar ataxia is characterized by rapid onset of symptoms, typically developing over a few hours but at most over one to two days. In cases associated with a prodromal illness, the ataxia typically follows it within days to three weeks. Gait disturbance is the primary symptom in most patients, but in other patients the cerebellar dysfunction may be limited to fine motor control problems or tremor. Associated symptoms may include nystagmus (present in roughly one-half of cases reported), slurred or garbled speech, vomiting, irritability, dysarthria or, in older children, headache. Fever, meningismus, and seizures are absent.

Acute Disseminated Encephalomyelitis - febrile illness in four weeks prior to neurological signs. Fever, headache, vomiting, and meningismus are often present at the time of initial presentation and may persist during the hospitalization. Encephalopathy is a characteristic feature and usually develops rapidly in association with multifocal neurologic deficits. New clinical symptoms may develop during hospitalization. Progression of neurologic signs to maximum deficits usually occurs over four to seven days. The level of consciousness ranges from lethargy to frank coma

66
Q

A 12-year-old boy is referred for evaluation of excessive daytime tiredness. He was recently found asleep in a school examination. There was no history of hyperphagia or sexual precocity. A polysomnogram did not reveal any evidence of obstructive sleep apnoea syndrome.
The most appropriate next step is:

A. a computed tomography (CT) scan of the brain.
B. a multiple sleep latency test.
C. electroencephalogram (EEG).
D. HLA typing.
E. psychological assessment.

A

B. Multiple sleep latency test.

When pt presents with excessive daytime sleepiness, narcolepsy must first be ruled out by multiple sleep latency test.

67
Q

An infant has sustained a large cerebral infarct involving most of the left occipital lobe. The remainder of the brain is unaffected. A later neurological examination would be most likely to reveal which one of the following?
A. Afferent pupillary defect.
B. Bitemporal hemianopia.
C. Central scotoma.
D. Right amblyopia.
E. Right homonymous hemianopia.

A

E. Right homonymous hemianopia.

68
Q

A 14-year-old girl is hiking on a school camp. She has been carrying a heavy backpack for about four hours during the day. She wakes the next morning with pain across her right shoulder and upper arm. The pain persists for about a day and she then develops arm weakness.
Examination shows weakness of abduction at the shoulder and of flexion at the elbow. She has a depressed biceps deep tendon reflex and sensory loss on the lateral aspect of her arm at the insertion of the deltoid muscle.
The most likely site of injury is to the:

A. accessory nerve.
B. axillary nerve.
C. brachial plexus.
D. C7/C8 spinal roots.
E. radial nerve.

A

C. brachial plexus.

Accessory Nerve - CN XI. M - SCM and trapezius.

Axillary Nerve - (C5, C6). M - glenohumeral joint, teres minor, deltoid. S - superolateral aspect of forearm.

Brachial plexus - (C5-T1) - consists of Musculocutaneous, Axillary, Radial, Median, Ulnar nerves.

C7/C8 spinal roots - M - extensors and flexors of the wrist

Radial Nerve - (C5-T1). M - posterior compartments arm and forearm. S - posterior and inferolateral arm, posterior forearm and dorsum of hand lateral to axial line of digit 4.

Biceps supplied by musculocutaneous nerve (C5, C6)

Dermatome at insertion of deltiod is C6.

69
Q

The patient shown in the photograph below is pushing with arms extended against a fixed object.

Which one of the following nerves is affected?
A. Accessory.
B. Long thoracic.
C. Pectoral.
D. Subclavian.
E. Suprascapular.

A

B. Long Thoracic Nerve

Image shows winged scapula. Abnormal position of scapula sitting away from thoracic wall. Paralysis of the serratus anterior due to injury of the long thoracic nerve.

Long Thoracic N - Serratus anterior M - Protracts scapula and holds it against thoracic wall, rotates scapula.

Accessory N - CN XI. Trapezius - Elevates, retracts and rotates scapula.

Pectoral N - Pectoralis major and minor. Stabilises scapula by drawing anteriorly and inferiorly.

Subclavian N - Subclavius M - Anchors and depresses clavicle.

Suprascapular N - Supraspinatus and infraspinatus M - Initiates and helps deltoid to abduct arm and acts with rotator cuff muscles (supra), laterally rotates arm, helps hold humeral head in glenoid cavity of scapula.

70
Q

The full term infant, shown above at age 36 hours, was born to a primigravida. The pregnancy was complicated by polyhydramnios and poor foetal movements. The infant required intubation and ventilation from birth for poor respiratory effort. He remained floppy and had few spontaneous movements.
Which one of the following is most likely to provide diagnostic information?
A. Creatine kinase levels.
B. Electromyography (EMG).
C. Magnetic resonance imaging (MRI) scan of the brain.
D. Muscle biopsy.
E. Triplet repeat studies.

A

E. Triplet repeat studies.

Pt has Congenial Myotonic Dystrophy.

Congenital myotonic dystrophy —

  • Only occurs in DM1
  • characterized by profound hypotonia, facial diplegia, poor feeding, arthrogryposis (especially of the legs), and respiratory failure.
  • Characteristic “V” shape of the upper lip that results from facial diplegia.
  • In some cases, DM1 may present before birth as polyhydramnios talipes and reduced fetal movement.
  • In the most severely affected infants, polyhydramnios is common during pregnancy and is related to disturbance in swallowing.
  • Labor also tends to be either prolonged or abbreviated, presumably on the basis of maternal uterine muscle involvement.

Myotonia is not usually present in the first year of life, and electrical myotonia is rare. Respiratory involvement is common and is the leading cause of death in the neonatal period. Mechanical ventilation is required for 70 to 80 percent or more of patients. Gastrointestinal and feeding difficulties are also common, with many children requiring a nasogastric or gastric feeding tube. With intensive support, most infants survive the neonatal period, but the overall mortality rate is approximately 15 to 20 percent, and approaches 40 percent in severely affected infants

Diagnosis

  • Gold standard is genetic testing for explanded CTG repeat (triplet repeat) of DMPK gene.
  • Role for muscle biopsy is limited. However, muscle biopsy may be useful to help distinguish DM2 from an inflammatory or metabolic myopathy when they cannot be differentiated by clinical presentation alone. Muscle biopsy may suggest the diagnosis of myotonic dystrophy in atypical cases with minimal weakness, unexplained elevations in CK and nonspecific EMG findings.
  • CK may be mild to moderately increased, but not specific.
  • EMG requires multiple fibres, not specific test.
  • MRI has no role.
71
Q

A nine-year-old girl presents to the emergency department following an episode of confusion. Her parents say that the episode began with a brief outburst of laughter while sitting at the dinner table, after which she was unable to respond, but picked at her food, chewing without food in her mouth as well as rocking backwards and forwards in her chair. She was taken to the couch where she lay down and slept for half an hour. After she woke she behaved normally, but was unable to describe what had happened. In the last year or so there have been several other episodes of laughter that have seemed ‘forced’ and ‘out of place’.

She has a past history of precocious puberty diagnosed at six years of age and treated with monthly injections of the luteinising hormone-releasing hormone (LHRH) agonist leuprolide.

Physical examination is normal. An electroencephalogram (EEG) is reported as showing some paroxysmal sharp-slow activity over the left temporal region. A magnetic resonance imaging (MRI) scan of the brain is performed and is shown below.

Which one of the following is the most likely diagnosis?
A. Colloid cyst of the third ventricle.
B. Hypothalamic hamartoma.
C. Mesial temporal sclerosis.
D. Pituitary adenoma.
E. Tuberous sclerosis.

A

B. Hypothalamic hamartoma

A hypothalamic hamartoma (HH) is a rare benign brain tumor or lesion of the hypothalamus. The hypothalamus is located at the base of the brain, and is responsible for many of the “automatic” functions of the brain including hunger, thirst, temperature, passion, and hormone regulation. A hypothalamic hamartoma can cause many types of seizures, precocious (premature) puberty, cognitive deterioration and severe behavioral difficulties known as rage behaviors.

There is tremendous variability in the symptoms experienced by individuals diagnosed with hypothalamic hamartomas (HH). However, many individuals experience some combination of the following:

  • Gelastic (laughing), dacrystic (crying) & other seizures
  • Cognitive impairments
  • Hypothalamic rages & other emotional and behavioral difficulties
  • Precocious puberty
  • Pallister-Hall syndrome

Leuprolide acts as an agonist at pituitary GnRH receptors. By interrupting the normal pulsatile stimulation of, and thus desensitizing, the GnRH receptors, it indirectly downregulates the secretion of gonadotropins luteinizing hormone (LH) and follicle-stimulating hormone (FSH), leading to hypogonadism and thus a dramatic reduction in estradiol and testosterone levels in both sexes.

72
Q

A seven-year-old boy has been awakening at 2 a.m. confused and crying with headache almost every night for the last two weeks. He has vomited on several of these occasions. He is well during the day. His mother has a history of migraine.

Clinical examination reveals a normally grown child with a normal gait and no evidence of papilloedema, nystagmus or localising neurological signs.

The most appropriate next step in management is to:

A. commence prophylactic propranolol.
B. counsel regarding night terrors.
C. image with computerised axial tomography.
D. perform an electroencephalogram (EEG).
E. reassure and invite to return if things get worse.

A

C. image with computerised axial tomography.

Not currently first line investigation, would now do MRI for headaches occurring at night to rule out tumour growth.

73
Q

A five-year-old girl is brought to her paediatrician because of recurrent headaches. During the physical examination a number of skin lesions were found. These are demonstrated in the photograph below.

Which of the following is the least likely to be associated with her condition?
A. Acoustic neuroma.
B. Learning difficulties.
C. Macrocephaly.
D. Pseudoarthrosis.
E. Scoliosis.

A

Acoustic Neuroma

Pt has NF1. Acoustic neuroma is a feature of NF2 (aka vestibular schwannoma), common feature. Benign but can cause significant morbidity. Not common finding in NF1.

74
Q

A 10-year-old girl presents with primary nocturnal and diurnal enuresis. She wakes herself twice a night to go to the toilet. If she sleeps through she is wet in the morning. Her underpants are often wet at the end of the day, and she says she does not feel herself urinating.
She has always walked on her toes. Her performance at school is satisfactory. Her physical examination shows a normal musculoskeletal system. Her gait is balanced when on tiptoe, but when asked to walk on flat feet she can only walk with a wide-based gait. Her Achilles tendons are tight, she has very brisk ankle jerks, 5 beats of ankle clonus bilaterally and bilateral upgoing plantar responses. Sensation is normal.
Her micturating cystourethrogram is shown below.

Which of the following is the most likely diagnosis?
A. Cerebral palsy.
B. Charcot-Marie-Tooth disease.
C. Friedreich ataxia.
D. Idiopathic toe-walker.
E. Spinal dysraphism.

A

E. Spinal dysraphism

Spinal Dysraphism - Spinal dysraphism is a medical term that refers to neurological disorders related to malformations of the spinal cord. Tethered spinal cord syndrome is a type of spinal dysraphism. During early development, a defect in the dura mater allows communication of the spinal cord with subcutaneous tissues, anchoring the conus medullaris and preventing the normal upward migration of the spinal cord.

Presentation:

  • childhood or in adulthood
  • pain in the lower extremities and perianal region,
  • progressive weakness
  • sensory disturbances
  • sphincter dysfunction
  • high-arched feet
  • discrepancy in muscle size and strength between the legs
  • gait abnormality
  • Because the cauda equina and conus medullaris are affected, both spastic and flaccid bowel and bladder dysfunction may be noted.
  • Cutaneous signs include sacral dimples, hair tufts.
75
Q

A three-year-old child with delayed language development, aggressive behaviour and seizures is brought to see you for evaluation. His mother is concerned about the increasing size of a ‘birth mark’ on his forehead, which is shown below.

What other findings are you most likely to find on clinical examination?
A. Adenoma sebaceum.
B. Axillary freckling.
C. Café-au-lait spots.
D. Depigmented patches.
E. Increased pigmentation in skin creases

A

D. Depigmented patches.
Pt has tuberous sclerosis complex. The patch on his forehead is a distinctive brown fibrous patch which is often the first recognised feature of TSC.

Skin findings:

  • Ash-leaf spots (hypopigmented)
  • Angiofibromas (malar regions of face)
  • Shagreen patches lower trunk

Other clinical features:

  • Seizures (most common and causes most morbidity)
  • Cognitive deficits and learning disabilities
  • Autism and behavioural problems
76
Q

A child born at term is brought to his paediatrician at three months of age because of daily clusters of brief forward jerks of the limbs, head and trunk. General examination is normal with the exception of moderate truncal hypotonia. A T2-weighted magnetic resonance imaging (MRI) scan is shown below.

Which of the following is the most likely diagnosis?
A. Hydrocephalus.
B. Krabbe disease.
C. Leigh disease.
D. Lissencephaly.
E. Tuberous sclerosis.

A

D. Lissencephaly

Lissencephaly, or agyria, is a rare disorder that is characterized by the absence of cerebral convolutions and a poorly formed sylvian fissure, giving the appearance of a 3-4 mo fetal brain. The condition is probably a result of faulty neuroblast migration during early embryonic life and is usually associated with enlarged lateral ventricles and heterotopias in the white matter.
Presentation:

  • failure to thrive
  • microcephaly
  • marked developmental delay
  • severe seizure disorder
  • Ocular abnormalities are common, including hypoplasia of the optic nerve and microphthalmia.

Hydrocephalus is not a specific disease; it represents a diverse group of conditions that result from impaired circulation and absorption of CSF or, in rare circumstances, from increased production of CSF by a choroid plexus papilloma.

In an infant, an accelerated rate of enlargement of the head is the most prominent sign. In addition, the anterior fontanel is wide open and bulging, and the scalp veins are dilated. The forehead is broad, and the eyes might deviate downward because of impingement of the dilated suprapineal recess on the tectum, producing the setting-sun eye sign. Long-tract signs including brisk tendon reflexes, spasticity, clonus (particularly in the lower extremities), and Babinski sign are common owing to stretching and disruption of the corticospinal fibers originating from the leg region of the motor cortex. In an older child, the cranial sutures are partially closed so that the signs of hydrocephalus may be subtler. Irritability, lethargy, poor appetite, and vomiting are common to both age groups, and headache is a prominent symptom in older patients. A gradual change in personality and deterioration in academic productivity suggest a slowly progressive form of hydrocephalus. With regard to other clinical signs, serial measurements of the head circumference often indicate an increased velocity of growth. Percussion of the skull might produce a cracked pot sound or MacEwen’s sign, indicating separation of the sutures.

Krabbe Disease is a rare autosomal recessive lysosomal storage disorder caused by the deficiency of galactocerebrosidase. Most patients with Krabbe disease present with symptoms within the first six months of life; approximately 10 percent present later in life, including adulthood.

A peripheral motor sensory neuropathy occurs in all patients, but the early onset forms are dominated by symptoms related to central nervous system dysfunction. Infantile onset disease is associated with irritability, developmental delay or regression, limb spasticity, axial hypotonia, absent reflexes, optic atrophy, and microcephaly.

Seizures and tonic extensor spasms eventually appear.

Typically there is rapid regression to a decerebrate condition and death before two years of age.

Leigh Disease is a subacute necrotising encephalomyopathy. In decreasing order of severity, the affected areas are the basal ganglia, brainstem cerebellum, and cerebral cortex .

Classic presentation:

  • infant who presents with central hypotonia, developmental regression or arrest, and signs of brainstem or basal ganglia involvement.
  • Diagnosis is usually confirmed by radiologic or pathologic evidence of symmetric lesions affecting the basal ganglia, brainstem, and subthalamic nuclei.
  • Patients with Leigh disease frequently present with developmental delay, seizures, altered consciousness, failure to thrive, pericardial effusion, and dilated cardiomyopathy.
  • The prognosis for Leigh syndrome is poor. In a study of 14 cases, there were 7 fatalities before the age of 1.5 yr.
  • Brain MRI shows abnormal white matter signal in the putamen, basal ganglia, and brainstem on T2 and FLAIR sequence images.

Tuberous Sclerosis - MRI findings in TSC include glioneuronal hamartomas (aka cortical tubers), subependymal nodules, subependymal giant cell tumours, and white matter abnormalities.

Skin findings - Hypomelanotic lesions (ash leaf spots), shagreen patch, angiofibromas/fibroadenomas, and brown fibrous plaque on forehead. Seizures occur in 80-90% pts and 60% develop by age 1 year. Infantile spasm most common type of seizure.

77
Q

A 15-year-old girl presents with a four-day history of increasing difficulty with vision. She initially noticed some pain around her right eye, then difficulty reading. Her symptoms progressed fairly rapidly over the next 48 hours to the point where she can only count fingers out of her right eye.
Examination reveals a right afferent pupillary defect, mild blurring of the right disk margin, and severely impaired vision. Her left eye is normal, as is the rest of her neurological examination.
Past history is significant for an episode of tinnitus and vertigo, which lasted for about one week one year ago, and an episode of numbness of her left arm lasting two weeks six months ago. Both these episodes resolved spontaneously.
A magnetic resonance imaging (MRI) scan of her brain is performed and is shown below.

Which one of the following is the most likely diagnosis?
A. Acute disseminated encephalomyelitis.
B. Multiple sclerosis.
C. Subacute sclerosing panencephalitis.
D. Systemic lupus erythematosus (SLE).
E. Toxoplasmosis.

A

B. Multiple Sclerosis

Multiple Sclerosis is a chronic demyelinating disorder of the brain, spinal cord and optic nerves characterized by a relapsing-remitting course of neurologic episodes separated in time and space. Only 2-5% of pts have first symptoms before age 18 years.

A complex interplay of environmental, infectious, and genetic factors influence MS susceptibility. Immune system dysregulation involving T and B lymphocytes triggers inflammation, axonal demyelination, axonal loss, and regeneration within both white and gray matter. Inflammatory infiltrates within actively demyelinating lesions of relapsing-remitting MS are targets for disease modifying therapy (DMT). Neurodegenerative changes predominate in progressive forms of MS.

Presenting symptoms in pediatric MS include hemiparesis or paraparesis, unilateral or bilateral optic neuritis, focal sensory loss, ataxia, diplopia, dysarthria, or bowel/bladder dysfunction.

Cranial MRI exhibits discrete T2 lesions in cerebral white matter, particularly periventricular regions as well as brainstem, cerebellum, and juxtacortical and deep gray matter.

ADEM is an initial inflammatory, demyelinating event with multifocal neurologic deficits, typically accompanied by encephalopathy.
Initial symptoms of ADEM may include lethargy, fever, headache, vomiting, meningeal signs, and seizure, including status epilepticus. Encephalopathy is a hallmark of ADEM, ranging from ongoing confusion to persistent irritability to coma. Focal neurologic deficits can be difficult to ascertain in the obtunded or very young child but common neurologic signs in ADEM include visual loss, cranial neuropathies, ataxia, motor and sensory deficits, plus bladder/bowel dysfunction with concurrent spinal cord demyelination.

Imaging - Head CT may be normal or show hypodense regions. Cranial MRI, the imaging study of choice, typically exhibits large, multifocal and sometimes confluent or tumefactive T2 lesions with variable enhancement within white and often gray matter of the cerebral hemispheres, cerebellum, and brainstem.

Subacute sclerosing panencephalitis - rare, progressive neurologic disorder caused by persistent measles virus infection of the CNS. Occurs in individuals who have been exposed to natural measles virus in early childhood.
Early Clinical manifestations:

  • Early in disease - personality changes, aggressive behavior, and impaired cognitive function
  • Myoclonic seizures

Later Clinical Manifestations:

  • Generalized tonic-clonic convulsions
  • hypertonia
  • choreoathetosis,
  • followed by progressive bulbar palsy, hyperthermia, and decerebrate postures.
  • papilledema in approximately 20% of the cases.
  • Optic atrophy, chorioretinitis, and macular pigmentation are observed in most patients.

The diagnosis by the typical clinical course and 1 of the following: (1) measles antibody detected in the CSF, (2) a characteristic electroencephalogram consisting of bursts of high-voltage slow waves interspersed with a normal background that occur with a constant periodicity in the early stages of the disease, and (3) typical histologic findings in the brain biopsy or postmortem specimen.
Treatment with a series of antiviral agents has been attempted without success.

Death occurs usually within 1-2 yr from the onset of symptoms.

Systemic lupus erythematosus (SLE) is a chronic autoimmune disease characterized by multisystem inflammation and the presence of circulating autoantibodies directed against self-antigens.
Most Common Clinical Manifestations:

  • fever
  • fatigue
  • hematologic abnormalities
  • arthralgia
  • arthritis
  • Renal disease in SLE is often asymptomatic (BP monitoring and urinalysis critical)
  • SLE is often characterized by periods of flare and disease quiescence or may follow a more smoldering disease course.

Toxoplasmosis -Toxoplasma gondii, an obligate intracellular protozoan, is acquired perorally, transplacentally, or, rarely, parenterally in laboratory accidents; by transfusion; or from a transplanted organ. In immunologically normal children, acute acquired infection may be asymptomatic, cause lymphadenopathy, or affect almost any organ. Once acquired, latent encysted organisms persist in the host throughout life. In immunocompromised infants or children, either initial acquisition or recrudescence of latent organisms often causes signs or symptoms related to the central nervous system (CNS).

If untreated, congenital infection often causes disease either perinatally or later in life, most frequently chorioretinitis and CNS lesions. Other manifestations such as intrauterine growth retardation, cognitive and motor deficits, fever, lymphadenopathy, rash, hearing loss, pneumonitis, hepatitis, and thrombocytopenia also occur. Congenital toxoplasmosis in infants with HIV infection may be fulminant.

78
Q

A previously well child presents with a two-day history of ataxia, vomiting and mild papilloedema.
A computed tomography (CT) scan of his brain is performed and is shown below.

Which one of the following is not seen in his scan?

A. Dilated 3rd ventricle.
B. Dilated 4th ventricle.
C. Dilated lateral ventricles.
D. Oedema of frontal horns.
E. Swollen cerebellum.

A

B. Dilated 4th Ventricle

79
Q

A mother brings her daughter to a paediatrician because of concerns about her daughter’s eyes (shown
above). The only significant past history is of a difficult delivery due to large gestational size.
Which one of the following is the most likely diagnosis?

A. Left Horner syndrome.
B. Left oculomotor nerve palsy.
C. Left proptosis.
D. Right congenital ptosis.
E. Right Horner syndrome.

A

D. Right congenital ptosis

Congenital Ptosis is most often associated with absence or reduction of striated levator palpebrae superioris muscle. Müller’s muscle remains relatively intact.
Congenital ptosis is usually unilateral (in 69 to 75 percent), neurologically isolated, and nonprogressive. There can be a familial association, but it may be unrecognized if family members are only mildly affected. Levator function (LF) varies in proportion to the degree of ptosis. Lid creases are often absent or lower than normal. Other ophthalmologic findings may include amblyopia and strabismus in 20 to 30 percent.

Horner Syndrome
The degree of anisocoria is more marked in the dark than in light. There is associated dilation lag, an asymmetry in pupillary redilation between the two eyes when the light source is moved away from the eye. The Horner’s pupil will redilate more slowly (by 15 to 20 seconds) than the normal pupil.

The ptosis is minor (less than 2 mm) and occurs as a result of paralysis of the Müller’s muscle, which is innervated by the sympathetic pathway. The lower as well as the upper lid is affected, producing the so-called “upside-down ptosis.” This further narrows the palpebral fissure. The levator palpebrae superioris is unaffected; weakness of this muscle produces the more profound upper lid ptosis seen in third cranial nerve palsies.
Anhidrosis is present in central or preganglionic (first or second-order) lesions. The sympathetic fibers responsible for facial sweating and vasodilation branch off at the superior cervical ganglion from the remainder of the oculosympathetic pathway; thus, anhidrosis is not a feature of postganglionic or third-order lesions. This sign is frequently not apparent to patients or clinicians.

In infants and children, impaired facial flushing (Harlequin sign) is often more apparent than anhidrosis.

Acute features of sympathetic disruption can also include ipsilateral conjunctival injection, nasal stuffiness, and increased near point of accommodation.

A congenital Horner’s syndrome should be suspected when anisocoria is associated with heterochromia (unequal iris color, with the affected iris being lighter). This occurs because formation of iris pigment in the first several months of age is under sympathetic control. This may only be apparent if the natural color is relatively dark.

Proptosis (exophthalmos) - Protrusion of the eye. It may be caused by shallowness of the orbits, as in many craniofacial malformations, or by increased tissue mass within the orbit, as with neoplastic, vascular, and inflammatory disorders. Ocular complications include exposure keratopathy, ocular motor disturbances, and optic atrophy with loss of vision.

Oculomotor Nerve Palsy (CN III) - Usually congenital, associated with birth trauma or developmental anomaly. Acquired 3rd nerve palsies in children can be an ominous sign and can indicate a neurologic abnormality such as an intracranial neoplasm or an aneurysm. Other less-serious causes include an inflammatory or infectious lesion, head trauma, postviral syndromes, and migraines.
Clinical signs: exotropia and a hypotropia, or downward deviation of the affected eye, as well as complete or partial ptosis of the upper lid.

This characteristic strabismus results from the action of the normal, unopposed muscles, the lateral rectus muscle, and the superior oblique muscle. If the internal branch of the 3rd nerve is involved, pupillary dilation may be noted as well. Eye movements are usually limited nasally in elevation and in depression. In addition, clinical findings and treatment may be complicated in congenital and traumatic cases of 3rd nerve palsy owing to misdirection of regenerating nerve fibers, referred to as aberrant regeneration. This results in anomalous and paradoxical eyelid, eye, and pupil movement such as elevation of the eyelid, constriction of the pupil, or depression of the globe on attempted medial gaze.

80
Q

A three-year-old boy is evaluated because of longstanding hypotonia, delayed motor milestones and
elevated creatine kinase (CK). Family history revealed that during an anaesthetic, his father developed fever and muscle rigidity. A muscle biopsy was performed and is shown below. The section is stained with NADH-trichrome.

Which one of the following is the most likely diagnosis?

A. Central core myopathy.
B. Duchenne muscular dystrophy.
C. Myotonia congenita.
D. Nemaline myopathy.
E. Spinal muscular atrophy.

A

Central core myopathy is autosomal dominant or recessive. Malignant hypertension occurs in all pts with central core myopathy, occurs rarely in Duchenne and other muscular dystophies and other myopathies. Infantile hypotonia, proximal weakness, muscle wasting, and involvement of facial muscles and neck flexors are the typical features. High incidence of cardiac abnormalities. The disease is characterized pathologically by central cores within muscle fibers in which only amorphous, granular cytoplasm is found with an absence of myofibrils and organelles. Histochemical stains show a lack of enzymatic activities of all types within these cores.

81
Q

A previously well child presents with a two-day history of ataxia, vomiting and mild papilloedema.
A computed tomography (CT) scan of his brain is performed and is shown below.

Which one of the following is not seen in his scan?

A. Dilated 3rd ventricle.
B. Dilated 4th ventricle.
C. Dilated lateral ventricles.
D. Oedema of frontal horns.
E. Swollen cerebellum.

A

B. Dilated 4th Ventricle

82
Q

A mother brings her daughter to a paediatrician because of concerns about her daughter’s eyes (shown
above). The only significant past history is of a difficult delivery due to large gestational size.
Which one of the following is the most likely diagnosis?

A. Left Horner syndrome.
B. Left oculomotor nerve palsy.
C. Left proptosis.
D. Right congenital ptosis.
E. Right Horner syndrome.

A

D. Right congenital ptosis

Congenital Ptosis

  • Absence or reduction of striated levator palpebrae superioris muscle. Müller’s muscle remains relatively intact.
  • Usually unilateral (in 69 to 75 percent), neurologically isolated, and nonprogressive.
  • There can be a familial association, but it may be unrecognized if family members are only mildly affected.
  • Lid creases are often absent or lower than normal. Other ophthalmologic findings may include amblyopia and strabismus in 20 to 30 percent.

Horner Syndrome

  • Anisocoria (unequal pupil size) with associated dilation lag, an asymmetry in pupillary redilation between the two eyes when the light source is moved away from the eye. The Horner’s pupil will redilate more slowly (by 15 to 20 seconds) than the normal pupil.
  • Ptosis of upper and lower lid Muller muscle
  • Anhidrosis in central or preganglionic lesions.
  • Impaired facial flushing (Harlequin sign)

A congenital Horner’s syndrome should be suspected when anisocoria is associated with heterochromia (unequal iris color, with the affected iris being lighter). This occurs because formation of iris pigment in the first several months of age is under sympathetic control. This may only be apparent if the natural color is relatively dark.

Proptosis (exophthalmos)

  • Protrusion of the eye.
  • It may be caused by shallowness of the orbits, as in many craniofacial malformations, or by increased tissue mass within the orbit, as with neoplastic, vascular, and inflammatory disorders.
  • Ocular complications include exposure keratopathy, ocular motor disturbances, and optic atrophy with loss of vision.

Oculomotor Nerve Palsy (CN III)

  • Usually congenital, associated with birth trauma or developmental anomaly.
  • Acquired 3rd nerve palsies in children can be an ominous sign and can indicate a neurologic abnormality such as an intracranial neoplasm or an aneurysm.
  • Other less-serious causes include an inflammatory or infectious lesion, head trauma, postviral syndromes, and migraines.
  • Clinical signs: exotropia (turned out) and a hypotropia (downward deviation) as well as complete or partial ptosis of the upper lid.

This characteristic strabismus results from the action of the normal, unopposed muscles, the lateral rectus muscle, and the superior oblique muscle. If the internal branch of the 3rd nerve is involved, pupillary dilation may be noted as well. Eye movements are usually limited nasally in elevation and in depression. In addition, clinical findings and treatment may be complicated in congenital and traumatic cases of 3rd nerve palsy owing to misdirection of regenerating nerve fibers, referred to as aberrant regeneration. This results in anomalous and paradoxical eyelid, eye, and pupil movement such as elevation of the eyelid, constriction of the pupil, or depression of the globe on attempted medial gaze.

83
Q

*** Remembered Q***

An 8-year-old girl presents to the Emergency Department with her first generalised seizure. She has a past history of recurrent urinary tract infections. She has been otherwise well. On history, she has had 24 hours of generalised abdominal pain, headache and vomiting.

On arrival in the ED she is irritable, and complaining of altered vision. On examination she is afebrile, has a blood pressure of 158 / 80, and has blurred optic
disc margins consistent with papilloedema.
An MRI of her brain is performed, and reveals a posterior encephalopathy as below. Immediately after the MRI she has a further generalised seizure, which is terminated with IV midazolam. After the seizure her blood pressure is 178 / 104.

What is the most appropriate next step in medical management of this child?
A. Captopril
B. Frusemide
C. Mannitol
D. Nifedipine
E. Sodium nitroprusside

A

D. Nifedipine

PRES - Posterior Reversible Encephalopathy Syndrome

most often occurs in the setting of hypertensive crisis, preeclampsia, or with cytotoxic immunosuppressive therapy.

Lower BP with easily titratable antihypertensive such as nifedipine or labetalol.

(per up to date)

Sodium nitroprusside can cause raised ICP along with hypotension.